SlideShare une entreprise Scribd logo
1  sur  89
Télécharger pour lire hors ligne
ENDOCRINOLOGY
Archer Online USMLE Reviews
www.ArcherReview.com
All rights reserved
Archer Slides are intended for use with Archer USMLE step 3 video
lectures. Hence, most slides are very brief summaries of the concepts
which will be addressed in a detailed way with focus on High-yield
concepts in the Video lectures.
Thyroid Disorders
Thyrotoxicosis
Subacute Thyroiditis
Thyroid Function Tests
Hashimatos Thyroiditis
Hypothyroidism
Thyrotoxic periodic paralysis
Thyroid Nodule and Approach
Thyroid Cancer
Thyroid Diagnostic Studies
 Thyroid Function Tests : TSH, Total T4, Free T4, T3
levels.
 Thyroid Binding Globulin
 Thyroid Uptake Scan – Radio active iodine uptake scan
( RAIU scan)
 Antibodies : Thyroid stimulating immunoglobulins,
Anti-microsomal antibodies, Anti-thyroid peroxidase
antibodies
 Thyroid ultrasound
 Thyroid Biopsy, FNAC
TSH
Obtain screening TSH in patients with conditions
that may be explained by or worsened by
hyperthyroidism :
 Unexplained weight loss
 Anxiety or sleep disturbance
 Tachycardia, including supraventricular
tachycardia and new onset atrial fibrillation
 Osteoporosis ( always suspect and rule out
hyperthyroidism especially when osteoporosis is
worsening despite adequate therapy in the elderly)
TSH
Obtain screening TSH in patients with conditions that
may be explained by or worsened by hypothyroidism :
 History of thyroid disease
 Autoimmune disease (R/O co-existent hashimatos)
 Unexplained depression
 Cognitive dysfunction ( dementia, delirium)
 Hypercholesterolemia ( can be secondary to
hyopthyroidism or can be worsended by it)
 Unexplained constipation
Screen in early pregnancy
TSH
 Preferred Screening test for suspected HYPO/HYPER Thyroidism
 Preferred Follow-up test for patients receiving therapy for either hypo or
hyperthyrodism or pts who had thyroid cancer therapy
 If TSH High  possible Primary hypothyroidism or rarely, secondary
hyperthyroidism
 If TSH low  possible Primary hyperthyroidism or rarely, secondary
hypothyroidism
 If TSH low in pt on treatment for hypothyroidism  reduce dose of
levothyroxine. ( usually in increments of 25mcg each time)
 If TSH high in a pt on treatment for hypothyroidism  Increase dose of
levothyroxine. ( usually in increments of 25mcg each time)
 If TSH low in pt on treatment for hyperthyroidism  Increase dose of
Propylthiouracil/ Methimazole ( or may indicate inadequate therapy)
 If TSH high in a pt on treatment for hyperthyroidism  Reduce dose of
Propylthiouracil/ Methimazole ( Indicates excess antithyroid therapy.
 If Radioiodine therapy was chosen, then aim is to make the pt Hypothyroid in
order to achieve good cure rates in Graves thyrotoxicosis  in that case, you
need to start levothyroxine post radioiodine therapy or post surgically)
TFTs – Further Approach
Obtain Total and Free T4
levels, ONCE TSH comes
abnormal
TFTs – Further Approach
 If TSH high but Free T4 low  primary hypothyroidism
 Usually Hashimatos thyroiditis.
 In countries where Iodine deficiency is a problem, this could be due to
endemic goiter. (urine iodine low)
 Iatrogenic Hypothyroidism due to Thyroidectomy or Radioiodine therapy
 Drug induced  Amiodarone, Lithium, Iodine {excess iodine
immediately inhibits both new hormone synthesis (by blocking
organification, known as the Wolf-Chaikoff effect) and the release of
thyroid hormone. It also decreases gland size and vascularity. This effect
is transient, however, lasting approximately 1-3 weeks}, Interferon,
Thionamides
 Late phase of Post-partum thyroiditis, Subacute thyroiditis, silent
thyroiditis
TFTs – Further Approach
 If TSH High but Free T4 also high 
secondary Hyperthyroidism ( pituitary
adenomas) - rare
 Get a Pituitary MRI
TFTs – Further Approach
 If TSH high but Free T4 normal  Subclinical
Hypothyroidism
 Subclinical Hypothyroidism :
 Treatment recommended only :
 If clinical symptoms are present
 If hyperlipidemia (high LDL)
 If patient has goiter or
 If TSH > 10mU/l , For TSH between 5 to 10  obtain
TPO antibodies and consider treatment if TPO antibodies
high)  In cases of subclinical hypothyroidism, presence
of anti-TPO antibodies predicts progression to overt
Hypothyroidism ( Hashimato’s)
TFTs – Further Approach
 If TSH low but Free T4 high  primary hyperthyroidism
 Usually Graves Thyrotoxicosis ( Thyroid scan – homogenous,
diffuse uptake)
 Toxic Multinodular Goiter ( Thyroid scan – Heterogenous)
 Toxic Solitary Adenoma
 Early phase of Subacute Thyroiditis/ Silent Thyroiditis ( No
Uptake on Thyroid scan)
 Post-partum Thyroiditis ( painless, may be followed by
hypothyroid phase)
 Iatrogenic Hyperthyroidism due to excess levothyroxine therapy in
a hypothyroid patient
 Surreptious intake of Thyroid hormones
 Struma Ovarii ( Increased pelvic uptake on whole body iodine
scan)
 Gestational Thyrotoxicosis ( Increased HCG is responsible,
Nuclear imaging contraindicated here)
 Amiodarone induced ( two types of thyrotoxicosis)
TFTs – Further Approach
 If TSH low but Free T4 also low  secondary
Hypothyroidism  ( pituitary adenomas
causing compression, craniopharyngioma) 
clues are headaches, visual field deficits,
ophthalmoplegia), pituitary/ hypothalamic
surgery, pituitary/ hypothalamic infection/
infarction  Next step, get a Pituitary MRI
TFTs – Futher Approach
 If TSH low but Free T4 normal  get T3 level.
 If T3 high  T3 toxicosis.
 If T3 normal  Subclinical Hyperthyroidism  Get
an RAIU scan to see if this is Graves or a Toxic
adenoma .
 Subclinical hyperthyroidism can be just followed if no
clinical symptoms and if there is no significant reason to
treat.
 Treatment of subclinical hyperthyroidism is recommended
 in cases of osteoporosis ( Thyroxine increases bone resorption)
 In atrial fibrillation
 in elderly due to risk of atrial fibrillation and osteoporosis.
(If Graves, thionamide or I131. If Toxic adenoma, I131 is
preferred.)
TFTs
 Remember any combinations of TSH/Free T4
can occur in severe nonthyroidal illness
depending on the phase of the illness ( critical
phase, recovery phase)  Euthyroid sick
syndrome . Some combinations
 low TSH, Normal free T4 ( may look
like subclinical hyperthyroidism)
 High TSH, Normal free T4 ( recovery
phase, may look like subclinical hypothyroidism)
 Your clue to answer questions in such situations
is the history recent or ongoing critical illness.
Thyrotoxicosis
Important topics:
Graves disease
Subacute thyroiditis
Post partum thyroiditis
Amiodarone induced
Endocrinology   archer step3 lectures
Classification of Thyrotoxicosis based on
RAIU Scan
High RAIU Low RAIU
Graves' disease Subacute thyroiditis
Toxic multinodular
goiter*
Painless thyroiditis (includes postpartum
thyroiditis)
Toxic adenoma* Chronic thyroiditis with transient thyrotoxicosis
HCG-mediated
(choriocarcinoma)
Iatrogenic/surreptitious thyroid hormone
TSH-secreting pituitary
tumor
Metastatic functional follicular thyroid cancer
(after thyroidectomy)
Amiodarone-induced thyroiditis
Struma ovarii (uptake in pelvis)
Grave’s Disease
 Features : Diffuse Goiter, Ophthalmopathy ( proptosis, extraocular muscle paralysis, peri-
orbital edema are specific. Lid-lag and stare can occur with any hyperthyroidism ) and
Dermopathy (pre-tibial myxedema) .
 Diagnosis : clinical features of hyperthyroidism ( palpitations, sweating, diarrhea, anxiety,
tremor, hairloss, weightloss, menorrhagia) + diffuse goiter, thyroid bruit/thrill can be heard
due to hyperdynamic circulation and increased thyroid vascularity.
 Pain usually absent in thyroid ( PAIN may be present in subacute thyroiditis)
 Always r/o subacute thyroiditis in differential
 Obtain RAIU scan  Graves shows homogeneous uptake where as no
uptake in subacute thyroiditis/ Surreptious intake of LT4. { Measure serum
thyroglobulin and ESR to distinguish subacute thyroiditis (elevated thyroglobulin
and ESR) from surreptitious thyroid hormone ingestion (suppressed thyroglobulin,
normal ESR)}. Painless thyroiditis has elevated thyroglobulin but normal ESR.
 TSH low, Free T4 high.
 Antibodies present  Thyroid stimulating antibodies ( TSIgs) are high and are
responsible for goiter and hyperthyroidism. Anti-TSH antibodies, Anti-TPO
antibodies.
 Can be associated with other autoimmune diseases  Vitilgo, pernicious anemia
Treatment Options
 Treatment options in Hyperthyroidism :
 Medications ( Antithyroid drugs - PTU, Thionamides i.e;
methimizole ) , B-Blockers ( propranolol, atenolol) for
symptoms. ( propranolol preferred because it can block T4 to T3
conversion in high doses)
 Raioactive iodine therapy (131 I)
 Surgery ( Thyroidectomy)
 Consider the following when selecting treatment for
thyrotoxicosis:
 Patient preference
 Patient's age and comorbidities
 Severity of thyrotoxicosis
 Goiter size
 Presence of ophthalmopathy (in Graves' disease patients)
Treatment for Thyrotoxicosis Due to Overproduction of Thyroid Hormone
Modality Advantages Disadvantages
Antithyroid drug
therapy
Least invasive
Least costly
Lower risk of permanent hypothyroidism
Potential beneficial immunomodulatory
effects
Adverse drug effects
Low permanent remission rate
(approximately 33%-50%)
Radioiodine Moderately fast reduction in thyroid
hormone levels
Less invasive than surgery
Permanent hypothyroidism likely
Requires delay in pregnancy (6-12 months)
and breastfeeding
May precipitate new or worsened
ophthalmopathy
Slight risk of thyroid storm after treatment (
in severe Hyperthyroidism, use pre-
treatment)
Thyroidectomy Rapid reduction in thyroid hormone levels
Allows concurrent removal of suspicious
nodules (if present)
Allows reduction in size of large goiters
Most invasive
Most costly
Permanent scar
Permanent hypothyroidism likely
Potential injury to parathyroids and recurrent
laryngeal nerve
Anti-thyroid drug therapy
 Propyl thiouracil :
 Typical starting dose 100mg tid
 Will need to discontinue 2 weeks prior to Radioiodine therapy because
it can lead to failure of the therapy as it inhibits roadioiodine uptake by
thyroid. ( REMEMBER METHIMIZOLE WILL NOT MUCH
AFFECT RADIOIODINE UPTAKE – So, it is the preferred therapy
to start if I131 therapy is planned but stop it 3 days prior to I131)
 LESS POTENT than Methimizole.
 Preferred drug in severe Hyperthyroidism because it can block T4 to T3
Conversion ( this effect not seen with methimazole)
 Side effects : Rash, Agranulocytosis, Hepatocellular necrosis
 Safe in pregnancy ( as it highly binds to plasma protein and very little
crosses placenta. There is NO risk of Aplasia Cutis). However, Fetal
hypothyroidism can occur in high doses due to transplacetal passage.
 MONITOR LFTS AND CBC  If pts have sorethroat/ infection etc
suggesting possibility of agaranulocytosis d/c PTU and get a blood
count
Anti-thyroid drug therapy
 Methimizole :
 More potent than PTU
 Typical starting dose is 30mg/d – preferred drug for most patients
because single daily dose (except for patients with allergy to
methimazole, who are pregnant, or have severe thyrotoxicosis or thyroid
storm)
 Contraindicated in pregnancy due to risk of Aplasia cutis and higher
transplacental spread than PTU.
 Not preferred in breast feeding ( PTU is preferred in breast feeding)
 Side effects : Rash, Agranulocytosis, cholestatic jaundice
 Good for pre-treatment prior to I-131 therapy. ( pre-treatment is used to
normalize thyroid function before the administration of radioiodine and
to attenuate potential exacerbations following ablative radioiodine
therapy.  recommended for the elderly and those with underlying
cardiac disease, who may be more vulnerable to worsening
thyrotoxicosis during I 131 therapy.). Stop pre-treatment with
methimazole 3 days prior to I131 to prevent treatment failure. Restart 3
days after giving I131 ( I131 takes 12 to 15 weeks to correct
hyperthyroidism – monitor free T4 after I131 Therapy)
 PTU is preferred over methimizole in severe hyperthyroidism.
 IF USING ANTITHYROID THERAPY as sole primary
therapy use for at least 12-18 months before tapering.
 When started, Thionamides can correct hyperthyroidism in
about 4 to 8 weeks ( response expected in 4 to 8 wks)
Antithyroid drug therapy
 For Grave’s disease patient getting only anti
thyroid drug therapy, factors favoring remission
are :
- Small goiter
- Low T3/T4 ratio
- Mild thyrotoxicosis
- Negative TSH-receptor antibody titers after
treatment
- No prior relapse
 So, antithyroid drug therapy can be a
preferred modaility in these scenarios.
Aplasia Cutis
 Absence of skin in the newborn
 Can occur with Methimizole use during
pregnancy.
 PIC : Aplasia Cutis of scalp
Radioiodine therapy
 Select a dose of radioiodine based on the size of the patient's
goiter and the result of the RAIU test.
 Do not use radioactive iodine in patients who are pregnant
(absolute contraindication) or breastfeeding.
 Pregnancy should be delayed four to six months after
radioiodine therapy.
 Radioiodine is better avoided in patients with Graves'
ophthalmopathy.
( may cause worsening of ophthalmopathy)
 After Radioiodine therapy
 mother should stay away from children at least for 7 days (
Mother need to Arrange for a care taker for baby).
 A I131 recepient must avoid contact with pregnant women,
avoid sexual contact for one week after the therapy
Radioiodine Therapy – Follow Up
 Hypothyroidism is inevitable in most cases. The risk of Hypothyroidism depends
on the dose of I131.
 About 2/3 of the patients develop hypothyroidism in the first year after I131.
Even if they do not develop hypothyroidism in the first year, the TSH should be
followed every 6 months forever after I131 therapy since the risk of developing
hypothyroidism persists for lifetime
 Get free T4 in the early period after I131 therapy not TSH. TSH can be
misleading in that it can remain low for about 4 to 6 weeks even when patient’s
free T4 returns to normal and patient is clinically euthyroid.
 I131 typically takes 12 to 15 weeks to correct hyperthyroidism. So, thionamide has
to be restarted 3 days after I131 and Free T4 and TSH carefully monitored. Once
there is evidence of euthyroid state, taper and stop thionamide. Follow TSH, if
hypothyroidism sets in  start Levothyroxine)
 Start Levothyroxine after getting TSH level and Free T4 if the levels are consistent
with hypothyroidism .
Surgery
 Especially preferred in :
 Goiter causing obstructive symptoms
 Pregnant women allergic to PTU or develop agranulocytosis on PTU ( Radio
iodine is not an option in pregnancy)
 Intolerance to thionamides; refusal to take antithyroid drugs or radioactive iodine
 Recurrence after a trial of thionamide therapy for Graves’ disease
 Large MULTI-NODULAR goiter
 Suspicion of malignancy.
 Patients should be made euthyroid with thionamide therapy prior to
elective surgery. Once a euthyroid state is achieved, oral iodine is given
7 to 10 days preoperatively to reduce the vascularity of the gland 
then total/ subtotal thyroidectomy.
 Complications : Hypothyroidism inevitable, Recurrent laryngeal nerve
palsy (1%) , Permanent hypoparathyroidism ( 1% risk) and 1% risk of
recurrent hyperthyroidism
 Admit pts to ICU for 24 hrs s/p surgery  Monitor calcium level
q6hrs in patents s/p thyroid surgery and replete calcium aggressively if
<7mg%
Follow-up
 In pts on long term antithyroid drug therapy
 Monitor for adverse effects of antithyroid drugs such as rash,
hepatic dysfunction, and agranulocytosis.
 Repeat thyroid function tests every 3 to 6 months for the first
year and then every 6 to 12 months after attaining normal
FT4 and TSH levels.
 Follow serum FT4 and TSH levels at 4-week intervals
to ensure adequate replacement therapy ( for
hypothyroidism) immediately after radioiodine therapy.
Q1
 A 58-year-old woman has anxiety, tremors, excessive sweating, palpitations, and
insomnia of approximately 1 month’s duration. Her medical history is unremarkable.
She has had no recent pregnancies or miscarriages. She has a modest, nontender goiter
and no exophthalmos. She takes no medications and has had no recent radiologic
procedures. The
 24-hour radioactive iodine uptake is 10% (normal, 20% to 35%).
 Laboratory Studies
 Erythrocyte sedimentation rate 8 mm/h
 Free thyroxine 3.5 ng/dL
 Thyroid-stimulating hormone < 0.01 μU/mL
 Thyroglobulin 45 ng/mL (normal, 2 to 20 ng/mL) ( do not confuse thyroglobulin with
thyroid binding globulin – thyroglobulin is the one present in the thyroid follicles and
is released out and elevated whenever there is “thyroidITIS”
 Antithyroperoxidase 26 (normal, <2 μU/mL)
What is the most likely diagnosis?
 ( A ) Struma ovarii
 ( B ) Recent imaging study with an iodinated contrast
 ( C ) Subacute thyroiditis
 ( D ) Surreptitious use of thyroid hormones
 ( E ) Painless/silent thyroiditis
 (F) Use of Amiodarone
Q2
 A 58-year-old woman presents to your office with increasing fatigue, memory loss,
and depression for the past one year. Her history is significant for hypertension and
she takes enalapril for it. Her family history reveals a brother who takes a medication
for his thyroid. On physical examination, she is 168 cm (66 in) tall and weighs 73 kg
(162 lb). Blood pressure is 152/88 mm Hg and heart rate is 86/min. HEENT exam is
nornal without any thyroid enlargement.
 Laboratory Studies
 Hematocrit 46%
 Plasma glucose 82 mg/dL
 Total cholesterol 255 mg/dL
 High-density lipoprotein cholesterol 49 mg/dL
 Low-density lipoprotein cholesterol 187 mg/dL
 Thyroid-stimulating hormone 12.2 μU/mL
 A free thyroxine level of 0.9 ng/dL.
 Which of the following statements is correct about levothyroxine replacement
therapy for this patient?
 ( A ) Her LDL will decrease.
 ( B ) There will be no improvement in her symptoms.
 ( C ) She is likely to lose weight.
 ( D ) She will be at an increased risk of atrial fibrillation
 ( E ) she will have an increased risk of osteoporosis
ANS.A
 This patient has subclinical hypothyroidism. Rx
is recommended if serum TSH levels greater
than 10 μU/mL. Also, Rx is recommended in
patients having symptoms and effects of
hypothyroidism like hyperlipidemia In such
cases, levothyroxine therapy reduces LDL
cholesterol levels and improves symptoms.
Q3
A 50 y/o HIV +ve man with a CD4 count of 200 prrsents with complaints of
fatigue, weightloss and dry cough. Cough has been for past 2 days and has
high grade fever. No chest pain but he is visibly short of breath. His ABGs
revealed hypoxemic respiratory failure with a po2 of 45. He is started on
Bactrim IV and steroids.
 Laboratory Studies reveal
 Free thyroxine 0.9 ng/dL
 Triiodothyronine 22 ng/dL
 Thyroid-stimulating hormone 0.3 μU/mL
 Which of the following statements about this patient’s condition is true?
 ( A ) Perform a radioactive iodine uptake test
 ( B ) Using T3 treatment will improve his condition and prognosis.
 ( C ) The lab abnormalities are due to decreased peripheral conversion of thyroxine to
triiodothyronine
 ( D ) Serum thyroid binding globulin is increased
 ( E ) Pituitary MRI should be obtained
Euthyroid Sick Syndrome
Euthyroid Sick Syndrome
 Euthyroid sick syndrome is seen in acutely and
critically ill patients.
 Occurs due to alterations in the levels of
circulating thyroid hormones that can occur in
severe nonthyroidal illnesses.
 Features: low T3, high reverse t3 normal T4,
and low TSH levels. TSH starts elevating to
hypothyroid levels during “recovery” phase of
non thyroidal illness and returns to normal with
complete recovery. But many combinations can
occur
Q4
 A 32-year-old woman is evaluated for recent onset of fatigue, palpitations,
profuse sweating, and emotional lability. She gave birth to her second child 8
weeks ago and is not breast feeding. On physical examination, her pulse rate
is 100/min, and she has mild lid lag, a fine hand tremor, and a slightly
enlarged, nontender thyroid gland. Radioactive iodine uptake is less than 1%
at 4 and 24 hours (normal, 20% to 35%). Laboratory test results include a
serum thyroid-stimulating hormone level of less than 0.03 μU/mL and a free
thyroxine level of 3.8 ng/dL.
 Which of the following is the optimal treatment for this patient?
 ( A ) Antithyroid drugs
 ( B ) Radioactive iodine
 ( C ) Thyroidectomy
 ( D ) Prednisone
 ( E ) ß-blockers
Key Point
 Women presenting with thyrotoxicosis are treated with beta
blockers to decrease palpitations tremors..
 Antithyroid medications are not used in the thyrotoxic phase as
since thyroid is not overactive.
 Post partum thyroiditis may be followed by a hypothyroid phase
( due to destruction – which may start 4-8 months post-partum
and may last 9-12 months) - treated with thyroid hormone
replacement  which should be later tapered off  It is always
important to monitor and taper off thyroid hormone after
postpartum thyroiditis, since 80% of patients will regain normal
thyroid function and not require chronic therapy.
IMP! – AIT
 This condition occurs in up to 10% of patients who use
amiodarone, which has a very high iodine content. Two subtypes
occur: type 1 AIT is caused by iodine overload and occurs
primarily in patients with underlying goiters; type 2 AIT is caused
by drug-induced thyroid follicular damage (thyroiditis).
 Both types are associated with a low 24-hour radioactive iodine
uptake. No tests reliably distinguish between the two subtypes,
although an underlying goiter and detectable radioactive iodine
uptake are more common in type 1
 AIT. Treatment of type 1 AIT consists of administering
thionamides, with or without potassium perchlorate
 Type 2 AIT may respond to corticosteroid therapy.
 Patients who do not respond to this treatment may require
plasmapheresis, dialysis, or thyroidectomy.
 A 70-year-old man is evaluated because of a 1-year history of progressive weakness,
weight loss, and hand tremors. For nearly 3 years, he has been treated with amiodarone
for paroxysmal atrial flutter. He has no goiter and no history of thyroid disease.
Thyroid scan shows scant, patchy tracer uptake. The 24-hour radioactive iodine uptake
is 2.7% at 6 hours and 4.1% at 24 hours (normal, 20% to 35%). The serum thyroid-
stimulating hormone level is <0.01 μU/mL and the serum free thyroxine level is 3.8
ng/dL.
 Which of the following statements is true about the effects of amiodarone on
this patient’s thyroid function?
 ( A ) Amiodarone can cause thyrotoxicosis by producing iodine overload or thyroiditis.
 ( B ) Amiodarone blocks peripheral thyroid hormone receptors.
 ( C ) Amiodarone increases peripheral conversion of thyroxine to triiodothyronine.
 ( D ) Amiodarone directly suppresses pituitary secretion of thyroid-stimulating
hormone.
 ( E ) Amiodarone increases serum thyroid hormone protein binding.
 Two subtypes of amiodarone-induced
thyrotoxicosis occur: type 1 is caused by iodine
overload and occurs primarily in patients with
underlying goiters; type 2 is caused by drug-
induced thyroid follicular damage (thyroiditis).
Hypothyroidism
Important concepts:
 Hashimato’s
 Lithium induced
Myxedema coma
Hashimato’S Thyroiditis
 Autoimmune thyroiditis associated with lymphocytic
infiltration of thyroid gland.  causes gland disruption,
initially may lead to hashitoxicosis and then
hypothyroidism
 Diagnosis : HIGH TSH, LOW FREE T4 and Elevated
TPO antibodies/ anti microsomal antibodies.
 TSIgs may be present too
 Diffuse Goiter on physical exam.
 Association with other autoimmune diseases must be
kept in mind  pernicious anemia, addisons disease
 Can be associated with B cell lymphomas
Treating Hypothyroidism
 Levothyroxine (LT4) drug of choice.
 In young patients  start initial full replacement dose
calculated as 1.6 µg/kg/d. ( 70kg man – 112mcg/d)
 In obese patients  calculate the initial dose using ideal
body weight  of course , you can follow-up serum TSH
6-8 weeks later and titrate the dose.
 In elderly patients > 60 yrs of age  start at dose 25 to 50
mcg and titrate every 6-8 wks untill TSH comes to desired
range.
 In those with KNOWN CAD/ hx of CHF and
arrhythmias  Start at 12.5 mcg to 25 mcg dose and then
titrate every 6-8 wks untill TSH comes to desired range.
 Ideal Body Weight (men) = 50 + 2.3 (
Height(in) - 60 )
 Ideal Body Weight (women) = 45.5 + 2.3 (
Height(in) - 60 )
Follow up – Treatment adequacy
 TSH is used to follow up treatment adequacy
 Levothyroxine has a t 1/2 of 7days. It takes a
drug four to five t1/2s to reach steady state 
which means DO NOT INCREASE LT4 dose
prior to 4 to 6 weeks if pt is not symptomatic.
 TSH level should be obtained at 6 – 8 weeks
after starting therapy  because it takes this
time for raised to TSH to normalize and also
this is the time at which you can properly assess
adequacy of LT4 therapy because LT4 reaches
steady state around this time.
Follow up – Treatment adequacy
 Use TSH levels as the guide to thyroid hormone dosage
requirements.
 Once treatment is started, monitor serum TSH levels
every 6 to 8 weeks and adjust the LT4 dose until the
TSH value is in the desired range.
 Once TSH reaches desired range, recheck the
TSH level 3 to 6 months later, and then if normal
recheck annually.
 If the TSH value is above the normal range, increase
the LT4 dose by 12.5 to 25 µg/d and recheck in 6 to 8
weeks.
 If the TSH value is low, decrease the LT4 dose by
12.5 to 25 µg/d and recheck in 6 to 8 weeks
Recognize Interactions!
 Counsel patients to take levothyroxine at the
same time each day and to avoid taking it within
4 hours of iron tablets, calcium supplements,
antacids, and fiber supplements ( can reduce
LT4 absorption)
Lithium Induced
 Li Can cause hypothroidism
 Rx  do not discontinue lithium, Start
levothyroxine and continue lithium therapy.
Myxedema Coma
 Severe hypothyroidism associated with altered mental
status/ delirium/ coma.
 Myxedema coma  usually found in elderly patients who
have untreated or inadequately treated hypothyroidism
and then develop a precipitating event. ( precipitating
event for myxedema coma in a pt with
untreated/inadequately treated hypothyroidism can be
SEPSIS, Surgery, Trauma, GI bleeding, Stroke, MI, CHF,
Sedative use)
 So, always evaluate the pt for precipitating factor also 
treat both myxedema coma as well as precipitant.
Myxedema coma
 Admit to ICU for careful monitoring and appropriate
treatment.
 Get Endocrine consult always.
 Obtain blood cultures, CT head, cardiac enzymes, EKG,
CHEM 18, CBC R/o preciptating factors
 TSH, FREE T4
 RX :
 Vital support : intubation if prolonged coma for airway protection, IV
fluids, hypothermia blanket.
 Levothyroxine intravenosly (500 µg, then 50-100 µg/d for myxedema
coma, for npo pts 80% of oral dose ) + hydrocortosine 100 q8hrs x
2days to address possible decreased adrenal reserve in myxedema.
 Also can use combination of LT4+LT3 for rapid T4 and T3 repletion
in myxedema coma
 Look for and treat any precipitating factor
Thyroid Nodule
Approach
Approach – Palpable Thyroid Nodule
 If thyroid nodule palpable
 Get TSH First.
 If High TSH – suggests cold nodule/ Hashimatos  Get FNAC ( some
recommend ultrasound as next step here because hashimatos may have
benign nodularity that regress with therapy and ultrasound will help to see
if there are suspicious features. If U/S suspicious, then FNAC is
recommended. This may be optimal approach because hurthle cells of
hashimatos may cause false positives on cytology if the FNAC is obtained
from such benign nodule). Treat with levothyroxine if overt hypothyroidism
or subclinical hypothyroidism that warrants treatment.
 If TSH normal – suggests cold nodule - get FNAC next
 If TSH low - suggests Hot nodule ( toxic adenoma) but not confirmative
(What if there is GRAVES in the surrounding tissue and this is a cold
nodule?)  so, next step get RAIU scan  if COLD nodule, get FNAC. If
RAIU scan shows Hot nodule treat with I131 ( if there is overt
hyperthyroidism from this toxic adenoma) or just observation.
 Cold nodules are more likely to be malignant when compared to hot nodules ( hot/
functioning nodule virtually rules out malignancy)
 A negative FNAC does not rule out cancer completely. So, if clinical suspicion for
cancer is high  consider surgical excision of the entire nodule. ( especially, when
histology revealed follicular adenoma – remember, a negative FNAC from one area
may not rule out possibility of follicular carcinoma in follicular adenoma )
Approach – Thyroid Incidentalomas
 Thyroid Incidentalomas – These are those nodules ( not the palpable
ones) detected on ultrasound such as ultrasound done for other
palpable thyroid abnormalities, during carotid artery imaging or
ultrasound done for hyperparathyroidism).
 FNAC indicated if
 Nodule > 10 mm in diameter.
 On ultrasound, if nodule has suspicious features of malignancy 
hypoechoic, microcalcifications, irregular shape, blurred margin or
increased vascularity
 If there are risk factors for thyroid cancer ( family history, childhood
neck irradiation)
After FNAC - Further Approach
 If FNAC is benign
 Cystic nodule – use percutaneous Ethanol injection to get rid of nodule
 Solid Nodule – use suppressive therapy with levothyroxine if there are no
contraindications ( Heart disease, old age). Suppressive therapy with LT4
aims to reduce TSH and there by, regress the nodule
 If FNAC is malignant  SURGERY
 If FNAC is follicular adenoma (benign)  get a thyroid
scan if not already done  If this is hot/functioning
nodule, it rules out malignancy – so, follow-up. If this is a
cold nodule, a negative FNAC from one area may not rule
out possibility of follicular carcinoma in follicular adenoma
– so, do Surgical excision of the entire nodule
(sub-total thyroidectomy) in those cases.
 If FNAC is non-diagnostic  repeat FNAC
Guidelines ( Source: NEJM)
Thyroid Cancer - Prevalence
Prevalence of thyroid cancer is higher in the
following groups and hence, high clinical suspicion
should be maintained if you find a thyroid nodule
in these groups ( i.e; warrants further evaluation of
the nodule):
 Children
 Adults less than 30 yrs or above 60 yrs of age
 History of head and neck radiation (eg: Hodgkins )
 Family history of thyroid cancer
Radiation Exposure – Follow up for
detecting thyroid cancer
 Guidelines are not clear.
 The incidence of thyroid cancer after radiation exposure is high for
about four decades after exposure and then it declines.
 If there is a history of significant radiation exposure such as (
therapeutic external beam radiation therapy in children, survivors of
Japan atomic bombing, exposure during chernobyl nuclear incident),
physical examination by thyroid palpation and ultrasound should be
obtained every three years to detect any nodules.
 If there is a palpable nodule in these patients, ultrasound must still
be obtained to detect any non palpable nodules ( because multiple
nodules are a common finding in radiation exposed patients)
 All palpable nodules and suspicious nodules on ultrasound, must
undergo FNAC.
Pheochromocytoma
Pheochromocytoma
Consider evaluating for Pheochromocytoma in patients with:
 Refractory hypertension
 Hypertension, accompanied by hyperadrenergic spells with:
 Nonexertional palpitations
 Diaphoresis
 Headache
 Tremor
 Pallor
 Family history of familial pheochromocytoma
 A genetic syndrome that increases the risk of pheochromocytoma, such as:
 Multiple endocrine neoplasia type 2
 von Hippel-Lindau disease - RCC
 Neurofibromatosis type 1
 History of gastric stromal tumor or pulmonary chondromas (Carney triad)
 An incidentally discovered adrenal mass ( Adrenal Incidentalomas)
Pheochromocytoma
 Episodic Hypertension is charecterestic but
ORTHOSTATIC HYPOTENSION can be
seen.
 Pheochromocytomas are 10% tumors  10%
malignant, 10% bilateral and 10% familial.
Pheochromocytoma - Diagnosis
 Best diagnostic test and high specificity (99%)  24 hr Urinary
Metanephrines ( metanephrines or normetanephrines above the
upper limit of normal in a patient not taking an interfering
medication and not physically stressed is consistent with
pheochromocytoma )
 Plasma Metanephrines can be used if 24 hr unrine collection is not
possible –specificity (88%) lower than 24 hr urine metanephrines
 Obtain CT Abdomen to r/o Adrenal mass only if biochemical tests
are positive. Very sensitive (96%) for pheochromocytoma ( adrenal
mass) but low specificity due to high prevalence of adrenal
incidentalomas.
 MIBG scan  Very specific for pheochromocytoma. Sensitivity
lower than CT scan. (Sensitivity only 80% and specificity 100%)
MIBG (Metaiodobenzylguanidine) scan is the best test if looking for
extra adrenal pheochromocytomas
 Other tests – Plasma catecholamines, Urine VMA
False + ves
Medications and activities that may increase measured levels of
catecholamines and metabolites ( False +ves for Metanephrines/
VMA)
 Tricyclic antidepressants
 Labetalol
 Levodopa
 Drugs containing catecholamines (e.g., decongestants)
 Amphetamines
 Buspirone (and most psychoactive agents)
 Sotalol
 Withdrawal from clonidine hydrochloride and other drugs
 Ethanol
 Acetaminophen and phenoxybenzamine (fractionated plasma
metanephrines)
 Physical stress (e.g., stroke, obstructive sleep apnea)
Treatment
 Acute Hypertensive crises  use IV Phentolamine,
Sodium nitroprusside or Nicardipine.
 Definitive treatment is SURGERY. Send for histopath
to r/o malignancy.
 REMEMBER TO USE BOTH ALPHA AND BETA
ADRENERGIC BLOCKING AGENTS PRIOR TO
SURGERY – to prevent hypertensive crises during
surgery. ( use phenoxybenzamine + atenolol)
 Do not use non selective beta blocker without alpha
blocker ( eg: propranolol alone)
 Understand that relatively short-acting, selective α1-
adrenergic receptor blockers (e.g., prazosin, terazosin,
doxazosin) may be inadequate for preoperative drug
preparation  so phenoxybenzamine preferred for pre-
op therapy as it is long acting.
Primary
Hyperparathyroidism
-Parathyroid Adenomas
-Parathyroid hyperplasia
Hypercalcemia
 Etiology
 Clinical features : bones, moans, stones, groans
 Investigations: Ca, Phos, EKG, PTH, 24 hr Urinary calcium excretion or spot
urine calcium/creatinine ratio ( R/o familial hypocalciuric hypercalcemia)
 Management:
 Criteria for surgery in primary hyperparathyroidism
 Sestamibi scan only if surgery is planned/indicated
 Hypercalcemic crisis management – ivf + lasix after volume repletion only
 Indications for corticosteroids : are useful for treating hypercalcemia caused
by vitamin D toxicity, certain malignancies (eg, multiple myeloma, lymphoma),
sarcoidosis, and other granulomatous diseases
 Cinacalcet (Sensipar) -- Directly lowers parathyroid hormone (PTH) levels by
increasing sensitivity of calcium sensing receptor on chief cell of parathyroid
gland to extracellular calcium. Also results in concomitant serum calcium
decrease  Indicated for hypercalcemia with parathyroid carcinoma.
Do not lower Calcium too much  Serum calcium reduction may cause lowered
seizure threshold, paresthesia, myalgia, cramping, and tetany;
Criteria for Surgery – Primary
hyperparathyroidism
 Serum total calcium level >12 mg per dL (3 mmol per L) at any time
 Hyperparathyroid crisis (discrete episode of life-threatening
hypercalcemia)
 Marked hypercalciuria (urinary calcium excretion more than 400 mg
per day)
 Nephrolithiasis
 Impaired renal function
 Osteitis fibrosa cystica
 Reduced cortical bone density (measure with dual x-ray
absorptiometry or similar technique)
 Bone mass more than two standard deviations below age-matched controls (Z
score less than 2)
 Classic neuromuscular symptoms
 Proximal muscle weakness and atrophy, hyperreflexia, and gait
disturbance
 Age younger than 50
 A 66-year-old asymptomatic woman is evaluated for a serum
calcium level of 11 mg/dL detected during a routine screening
examination. Subsequently, the ionized calcium level is 5.7
mg/dL, and the intact parathyroid hormone level is elevated.
 Which of the following is the most appropriate next step in
the management of this patient?
 ( A ) Parathyroidectomy
 ( B ) Sestamibi scan of the parathyroid glands
 ( C ) Creatinine clearance and 24-hour urine calcium excretion
 ( D ) Chest radiograph
 ( E ) Estrogen replacement therapy
Hyperprolactinemia
-Stress
-Drugs (<100)
-Pituitary Tumors
-Stalk effect
-Hypothyroidism
Remember!
 “If suspecting Prolactin secreting adenomas or
while working up the cause of
hyperprolactinemia, the LEVEL OF
PROLACTIN is the best clue to the possible
etiology of hyperprolatinemia”
 Prolactin level usually less than 100 with drugs
or stress!
Prolactinomas
 Microademomas  prolactin levels usually > 100
 Macroadenomas  Prolactin levels usually very high
500 to 1000. Be aware of HOOK Effect.
 MRI is the next test if high prolactin level encountered
and cannot be explained by meds/ stress. MRIs do not
r/o small microadenomas (<3mm)
 For microadenomas/ non invasive macroadenomas 
use Bromocriptine ( dopamine agonist)  Consider
surgery for those patients not responding to or
intolerant of dopamine agonists.
 If Prolactin Producing Macroadenomas with mass
effect  Try bromocriptine first  if no response,
Surgery ( trans sphenoidal resection)
Pituitary Adenomas
 In deciding how to approach INCIDENTALLY
discovered pituitary tumors, first rule out Function
 R/O functioning adenomas first  prolactin level,
Dexamethasone suppression test, ACTH level, TSH
and IGF 1 level depending on the clinical features.
 Except Prolactinomas, all other functioning adenomas
are treated primarily by surgery
 If Non functioning adenomas  Surgery indicated only
if mass effects  Visual field defects , Headaches and
Hypopituitarism
 Follow patients with non functioning microadenomas
and small, noninvasive macroadenomas with a 6-month
MRI scan and yearly thereafter  If there is no
evidence of enlargement after 3 to 5 years, then
continue to follow patients clinically.
 A 25-year-old woman is evaluated because of a 9-month history of weight gain,
fatigue, muscle weakness, and depression. She bruises and bleeds easily, and her
menses have been irregular. Her medical history and family history are unremarkable.
On physical examination, she is 157 cm (62 in) tall and weighs 74 kg (164 lb). Blood
pressure is 160/95 mm Hg and pulse rate is 84/min. She has a rounded, plethoric face.
Her supraclavicular fat pads are notably full, and she has a mildly enlarged dorsal fat
pad (buffalo hump). She has several violaceous striae on the lower abdomen
and bruises on the left arm from recent blood testing.
 Laboratory Studies
 Urine cortisol 318 μg/24 h
 Morning serum cortisol 28 μg/dL
 Morning plasma adrenocorticotropic hormone 45 pg/mL
 After administration of dexamethasone, 8 mg orally at bedtime, the morning serum
cortisol level is 3 μg/dL. Findings on chest radiograph are normal.
 Which of the following tests should be ordered next?
 ( A ) Low-dose (1 mg) overnight dexamethasone suppression test
 ( B ) Magnetic resonance imaging scan of the pituitary gland
 ( C ) Computed tomography scan of the adrenal glands
 ( D ) Computed tomography scan of the lungs
 ( E ) Inferior petrosal sinus sampling
CARCINOID
-Flushing, diarrhea, wheezing are clinical features
-Ileal carcinoids commonly metastisizes to liver
-Appendiceal carcinoids are most common.
-If pt develops above symptoms on small doses of SSRIs
 suspect Carcinoid syndrome
-Screening test  Urine 5-HIAA
-Confirmatory test – Octreotide Scan
-Rx – medically, octreotide for symptoms
-Surgery is definitive if no metastases
Hypoglycemia
 Medication induced in Diabetics
Surreptious insulin/ sulfonyl urea abuse
Insulinoma
Treatment
INSULINOMA
- Whipples Triad – the symptoms of
hypoglycemia
- C-peptide level.
 A 38-year-old woman is evaluated while in a stuporous state. The patient is slender
and nearly comatose. She responds minimally to a loud voice and sternal pain. On
physical examination, the pulse rate is 110/min and blood pressure is 115/70 mm Hg.
An accompanying friend informs you that the patient is a nurse and that she has had
recent psychiatric problems. The blood glucose level is 14 mg/dL. Additional blood is
drawn, and the patient is quickly resuscitated with intravenous glucose. A sulfonylurea
screen is negative.Laboratory Studies
 Serum calcium 9.5 mg/dL
 Serum insulin 45 mU/mL
 C-peptide 4.2 ng/mL (normal, 0.5 to 2.0 ng/mL)
 Proinsulin 0.6 ng/mL (normal, 0 to 0.2 ng/mL)
 Which of the following is the best explanation for these findings?
 ( A ) Solitary insulin-producing pancreatic islet cell tumor
 ( B ) Surreptitious insulin use
 ( C ) Surreptitious metformin use
 ( D ) Multiple endocrine neoplasia type 1
 ( E ) Multiple endocrine neoplasia type 2A
 The differential diagnosis of fasting
hypoglycemia associated with hyperinsulinemia
includes insulinoma, surreptitious insulin use,
and oral sulfonylurea ingestion.
 Surreptitious insulin use is associated with low
serum C-peptide and proinsulin levels.
Glucagonoma
Necrotizing migratory erythema
Octreotide scan
Congenital Adrenal
Hyperplasia
21 Hydroxylase deficiency
Increased 17 hydroxy progesterone, increased adrenal
androgens. Low cortisol
Ambiguous Genitilia in Congenital form
Needs Mineralocorticoid and Glucocorticoid replacement –
prevent salt wasting and hypotension
Adult onset CAH is possible in mild deficiency – can be
confused with PCOS
Polycystic Ovary
Disease
Conns Syndrome
Primary Hyperaldosteronism – Hypertension, Hypokalemia
and Alkalosis
PAC/ PRA ratio best screening test
CT scan to r/o adrenal mass after biochemical evidence.
Rx – medically with Spironolactone
Surgery if tumor > 4cm or suspicious features of adrenal
cancer or if refractory HTN despite maximal medical therapy
Cushings Syndrome
Etiology :Adrenal Cushings
Pituitary Cushings
Ectopic Cushings ( Small cell ca)
 Diagnosis : Overnight Low dose (1mg)
dexamethasone suppression test, Urinary 24 hr free
cortisol, High dose (8mg) dexamethasone suppression
test, ACTH level.
CT abdomen if Adrenal cushing suspected
Pituitary MRI if Pituitary adenoma suspected
False +ve Low Dose dexamathasone
test
Factors that can produce false-positive or false-negative
dexamethasone suppression tests:
 Alcohol, rifampin, phenytoin, and phenobarbital induce
the cytochrome P450-related enzymes and enhance
dexamethasone clearance  Cause False +ve
 Hepatic failure retards dexamethasone clearance 
cause false –ve.
 In renal failure, serum cortisol may appear
nonsuppressible by dexamethasone
 Obesity and depression give false-positive results
 Thiazolidinediones can give false-negative results
Addisons disease/
Adrenal insufficiency
-Random cortisol level
-Cosyntropin Stimulation test – 1hr and 24hr
stimulation tests
-Steroid replacement 
HYDROCORTISOME 100 Q8hrs +
Mineralocortcoid
Adrenal Incidentalomas
Work-up
 In all Adrenal incidentalomas, R/o functioning
adenomas first
 Do Low dose dexamethasone test (1mg) and Plasma
metanephrines in all patients
 Do PAC/PRA only if HTN is present or if serum
potassium is low.
 If Functioning adenomas – Rx is usually surgery
( except in Conns Syndrome where medical Rx
can be tried first)
Work-up
 For non-functioning adenomas, further
treatment depends on the size of adrenal mass.
 < 4cm  low risk of cancer ( 2% risk) . Follow-up
Ct scan at 6-12 months. If no change, no further
follow up.
 4cm-6cm  6% risk of cancer  Adrenalectomy or
a Follow-up CT Scan at 6months. If no growth, just
follow clinically. ( no more imaging)
 >6cm  high risk of malignancy (25% cases). Rx
with adrenalectomy
 After a fall while horseback riding, a 37-year-old woman is evaluated with an
emergency CT scan of the abdomen. The CT scan shows no evidence of a
ruptured spleen, but shows a 2.5-cm right adrenal mass. The patient’s medical
history, including review of systems, is normal. Findings on physical
examination, including blood pressure, are unremarkable. Plasma glucose
level, serum electrolyte levels, and renal function are normal.
 What is the most appropriate next step in the management of this
patient?
 ( A ) Surgical removal of the mass
 ( B ) MRI of the adrenal glands
 ( C ) Fine-needle aspiration biopsy of the mass under CT guidance
 ( D ) Repeated CT scan in 3 to 6 months
 ( E ) 24-Hour urinary catecholamines, metanephrine, and cortisol
 35-year-old man with a 10-year history of type 1
diabetes mellitus is evaluated because of recent onset of
morning hyperglycemia. For the past 10 days, his
morning blood glucose levels ranged from 220 to 300
mg/dL. He has also experienced nightmares recently.
 Which of the following best explains this patient’s
morning hyperglycemia?
 ( A ) Diabetic nephropathy
 ( B ) Undertreatment with insulin
 ( C ) Overtreatment with insulin
 ( D ) Diabetic neuropathy
KEY POINT
SOMGYI EFFECT
Nightmares are a clue and signifies a drop in
blood glucose to low levels
Next step  reduce the dose of pre-dinner
insulin

Contenu connexe

Tendances

Hyperthyroidism Guidelines
Hyperthyroidism GuidelinesHyperthyroidism Guidelines
Hyperthyroidism GuidelinesAlaa Mostafa
 
Thyroid Overview
Thyroid OverviewThyroid Overview
Thyroid OverviewMiami Dade
 
Hashimoto’s thyroiditis
Hashimoto’s thyroiditisHashimoto’s thyroiditis
Hashimoto’s thyroiditisPritesh Shukla
 
Thyroid disease in dental
Thyroid disease in dentalThyroid disease in dental
Thyroid disease in dentalMary Nasr
 
Hyperthyroidism & Anaesthetic Implications
Hyperthyroidism & Anaesthetic ImplicationsHyperthyroidism & Anaesthetic Implications
Hyperthyroidism & Anaesthetic ImplicationsDr.Daber Pareed
 
management of Hyperthyroidism
management of Hyperthyroidism  management of Hyperthyroidism
management of Hyperthyroidism Zelalem Semegnew
 
2012 Clinical Practice guidelines for hypothyroidism in adults: American Asso...
2012 Clinical Practice guidelines for hypothyroidism in adults: American Asso...2012 Clinical Practice guidelines for hypothyroidism in adults: American Asso...
2012 Clinical Practice guidelines for hypothyroidism in adults: American Asso...Jibran Mohsin
 
Thyroid storm
Thyroid stormThyroid storm
Thyroid stormdocgeero
 
Thyroid dysfunction and its management in dental office
Thyroid dysfunction and its management in dental officeThyroid dysfunction and its management in dental office
Thyroid dysfunction and its management in dental officeShankar Hemam
 
Clinical Practice Guidelines for hypothyroidism in adults: AACE and ATA 2012
Clinical Practice Guidelines for hypothyroidism in adults: AACE and ATA 2012Clinical Practice Guidelines for hypothyroidism in adults: AACE and ATA 2012
Clinical Practice Guidelines for hypothyroidism in adults: AACE and ATA 2012Jibran Mohsin
 
diagnosis and treatment of hypothyroidism
diagnosis and treatment of hypothyroidism diagnosis and treatment of hypothyroidism
diagnosis and treatment of hypothyroidism Balqees Majali
 
Hyperthyroidism Management
Hyperthyroidism ManagementHyperthyroidism Management
Hyperthyroidism ManagementArslan Tahir
 

Tendances (20)

Hyperthyroidism Guidelines
Hyperthyroidism GuidelinesHyperthyroidism Guidelines
Hyperthyroidism Guidelines
 
Hyperthyroidism
HyperthyroidismHyperthyroidism
Hyperthyroidism
 
Thyroid Overview
Thyroid OverviewThyroid Overview
Thyroid Overview
 
Hashimoto’s thyroiditis
Hashimoto’s thyroiditisHashimoto’s thyroiditis
Hashimoto’s thyroiditis
 
anaesthetic consideration for thyroid surgery
anaesthetic consideration for thyroid surgeryanaesthetic consideration for thyroid surgery
anaesthetic consideration for thyroid surgery
 
Thyroid disorders
Thyroid disordersThyroid disorders
Thyroid disorders
 
Thyroid disease in dental
Thyroid disease in dentalThyroid disease in dental
Thyroid disease in dental
 
Hyperthyroidism & Anaesthetic Implications
Hyperthyroidism & Anaesthetic ImplicationsHyperthyroidism & Anaesthetic Implications
Hyperthyroidism & Anaesthetic Implications
 
management of Hyperthyroidism
management of Hyperthyroidism  management of Hyperthyroidism
management of Hyperthyroidism
 
Hyperthyrodism
HyperthyrodismHyperthyrodism
Hyperthyrodism
 
2012 Clinical Practice guidelines for hypothyroidism in adults: American Asso...
2012 Clinical Practice guidelines for hypothyroidism in adults: American Asso...2012 Clinical Practice guidelines for hypothyroidism in adults: American Asso...
2012 Clinical Practice guidelines for hypothyroidism in adults: American Asso...
 
Updates in The Management of Thyroid Diseases
Updates in The Management of Thyroid DiseasesUpdates in The Management of Thyroid Diseases
Updates in The Management of Thyroid Diseases
 
Thyroid storm
Thyroid stormThyroid storm
Thyroid storm
 
Thyroid dysfunction and its management in dental office
Thyroid dysfunction and its management in dental officeThyroid dysfunction and its management in dental office
Thyroid dysfunction and its management in dental office
 
Clinical Practice Guidelines for hypothyroidism in adults: AACE and ATA 2012
Clinical Practice Guidelines for hypothyroidism in adults: AACE and ATA 2012Clinical Practice Guidelines for hypothyroidism in adults: AACE and ATA 2012
Clinical Practice Guidelines for hypothyroidism in adults: AACE and ATA 2012
 
Medical management of Thyroid disease
Medical management of Thyroid diseaseMedical management of Thyroid disease
Medical management of Thyroid disease
 
Thyroid diseases
Thyroid diseasesThyroid diseases
Thyroid diseases
 
diagnosis and treatment of hypothyroidism
diagnosis and treatment of hypothyroidism diagnosis and treatment of hypothyroidism
diagnosis and treatment of hypothyroidism
 
Thyrotoxicosis
ThyrotoxicosisThyrotoxicosis
Thyrotoxicosis
 
Hyperthyroidism Management
Hyperthyroidism ManagementHyperthyroidism Management
Hyperthyroidism Management
 

Similaire à Endocrinology archer step3 lectures

Similaire à Endocrinology archer step3 lectures (20)

Thyroidstorm ppt
Thyroidstorm pptThyroidstorm ppt
Thyroidstorm ppt
 
Thyroid
ThyroidThyroid
Thyroid
 
Thyroid
ThyroidThyroid
Thyroid
 
Thyroid
ThyroidThyroid
Thyroid
 
Thyroid
ThyroidThyroid
Thyroid
 
Thyroid disease and pregnancy ppt
Thyroid disease and pregnancy pptThyroid disease and pregnancy ppt
Thyroid disease and pregnancy ppt
 
Hyperthyroidism approach to management- dr selim
Hyperthyroidism approach to management- dr selimHyperthyroidism approach to management- dr selim
Hyperthyroidism approach to management- dr selim
 
MATERIAL.ppt
MATERIAL.pptMATERIAL.ppt
MATERIAL.ppt
 
Thyroid Drugs2[1]
Thyroid Drugs2[1]Thyroid Drugs2[1]
Thyroid Drugs2[1]
 
Thyroid Storm
Thyroid StormThyroid Storm
Thyroid Storm
 
THROID CRISIS.pptx
THROID CRISIS.pptxTHROID CRISIS.pptx
THROID CRISIS.pptx
 
Thyroid disease - A medusa of sorts
Thyroid disease - A medusa of sortsThyroid disease - A medusa of sorts
Thyroid disease - A medusa of sorts
 
Hypothyroidism
HypothyroidismHypothyroidism
Hypothyroidism
 
Thyroid Diseases & Its Information
Thyroid Diseases & Its InformationThyroid Diseases & Its Information
Thyroid Diseases & Its Information
 
Hyperthyroidism bsc nursing
Hyperthyroidism bsc nursingHyperthyroidism bsc nursing
Hyperthyroidism bsc nursing
 
Class thyroid and antithyroid drugs
Class thyroid and antithyroid drugsClass thyroid and antithyroid drugs
Class thyroid and antithyroid drugs
 
Pituitary Adrenal Thyroid Axis
Pituitary Adrenal Thyroid AxisPituitary Adrenal Thyroid Axis
Pituitary Adrenal Thyroid Axis
 
Hyperthyroidism.pptx
Hyperthyroidism.pptxHyperthyroidism.pptx
Hyperthyroidism.pptx
 
2727_Management of Thyroid Disorders.ppt
2727_Management of Thyroid Disorders.ppt2727_Management of Thyroid Disorders.ppt
2727_Management of Thyroid Disorders.ppt
 
02 Thyroid Hormones.ppt
02 Thyroid Hormones.ppt02 Thyroid Hormones.ppt
02 Thyroid Hormones.ppt
 

Plus de Archer Review USMLE and NCLEX

Safety and infection control - Archer NCLEX webinars
Safety and infection control - Archer NCLEX webinarsSafety and infection control - Archer NCLEX webinars
Safety and infection control - Archer NCLEX webinarsArcher Review USMLE and NCLEX
 
Archer NCLEX Webinars - Diets and Nutrition crash course
Archer NCLEX Webinars - Diets and Nutrition crash courseArcher NCLEX Webinars - Diets and Nutrition crash course
Archer NCLEX Webinars - Diets and Nutrition crash courseArcher Review USMLE and NCLEX
 
Archer NCLEX Rapid Review - Full Content Syllabus - Two days
Archer NCLEX Rapid Review - Full Content Syllabus - Two daysArcher NCLEX Rapid Review - Full Content Syllabus - Two days
Archer NCLEX Rapid Review - Full Content Syllabus - Two daysArcher Review USMLE and NCLEX
 
Cardiology and EKGs - Archer NCLEX crash course/ webinar
Cardiology and EKGs - Archer NCLEX crash course/ webinarCardiology and EKGs - Archer NCLEX crash course/ webinar
Cardiology and EKGs - Archer NCLEX crash course/ webinarArcher Review USMLE and NCLEX
 
Fluids and electrolytes Archer NCLEX content review webinar
Fluids and electrolytes  Archer NCLEX content review webinarFluids and electrolytes  Archer NCLEX content review webinar
Fluids and electrolytes Archer NCLEX content review webinarArcher Review USMLE and NCLEX
 
Infectious disease Archer USMLE Step3 Review Lectures
Infectious disease   Archer USMLE Step3 Review LecturesInfectious disease   Archer USMLE Step3 Review Lectures
Infectious disease Archer USMLE Step3 Review LecturesArcher Review USMLE and NCLEX
 

Plus de Archer Review USMLE and NCLEX (20)

Endocrine Archer NCLEX course Webinar
Endocrine Archer NCLEX course WebinarEndocrine Archer NCLEX course Webinar
Endocrine Archer NCLEX course Webinar
 
Safety and infection control - Archer NCLEX webinars
Safety and infection control - Archer NCLEX webinarsSafety and infection control - Archer NCLEX webinars
Safety and infection control - Archer NCLEX webinars
 
Archer NCLEX Webinars - Diets and Nutrition crash course
Archer NCLEX Webinars - Diets and Nutrition crash courseArcher NCLEX Webinars - Diets and Nutrition crash course
Archer NCLEX Webinars - Diets and Nutrition crash course
 
Rapid Content Prep - NCLEX- Archer Review
Rapid Content Prep - NCLEX- Archer ReviewRapid Content Prep - NCLEX- Archer Review
Rapid Content Prep - NCLEX- Archer Review
 
Archer NCLEX Rapid Review - Full Content Syllabus - Two days
Archer NCLEX Rapid Review - Full Content Syllabus - Two daysArcher NCLEX Rapid Review - Full Content Syllabus - Two days
Archer NCLEX Rapid Review - Full Content Syllabus - Two days
 
Cardiology and EKGs - Archer NCLEX crash course/ webinar
Cardiology and EKGs - Archer NCLEX crash course/ webinarCardiology and EKGs - Archer NCLEX crash course/ webinar
Cardiology and EKGs - Archer NCLEX crash course/ webinar
 
Fluids and electrolytes Archer NCLEX content review webinar
Fluids and electrolytes  Archer NCLEX content review webinarFluids and electrolytes  Archer NCLEX content review webinar
Fluids and electrolytes Archer NCLEX content review webinar
 
Archer NCLEX Pharmacology crash course - Webinar
Archer NCLEX Pharmacology crash course - WebinarArcher NCLEX Pharmacology crash course - Webinar
Archer NCLEX Pharmacology crash course - Webinar
 
Archer NCLEX Review - Electrolyte imbalances
Archer NCLEX Review - Electrolyte imbalancesArcher NCLEX Review - Electrolyte imbalances
Archer NCLEX Review - Electrolyte imbalances
 
Archer USMLE Step 3 Nephrology 2019
Archer USMLE Step 3 Nephrology 2019Archer USMLE Step 3 Nephrology 2019
Archer USMLE Step 3 Nephrology 2019
 
Pulmonology critical care- archer step3 lectures
Pulmonology  critical care- archer step3 lecturesPulmonology  critical care- archer step3 lectures
Pulmonology critical care- archer step3 lectures
 
Infectious disease Archer USMLE Step3 Review Lectures
Infectious disease   Archer USMLE Step3 Review LecturesInfectious disease   Archer USMLE Step3 Review Lectures
Infectious disease Archer USMLE Step3 Review Lectures
 
Archer step 3 ccs workshop 2019
Archer step 3 ccs workshop 2019Archer step 3 ccs workshop 2019
Archer step 3 ccs workshop 2019
 
Archer USMLE Step 3 CCS workshop 2018
Archer USMLE Step 3 CCS workshop 2018Archer USMLE Step 3 CCS workshop 2018
Archer USMLE Step 3 CCS workshop 2018
 
Cutaneous lymphomas
Cutaneous lymphomasCutaneous lymphomas
Cutaneous lymphomas
 
Acute Lymphoblastic Lymphoma
Acute Lymphoblastic LymphomaAcute Lymphoblastic Lymphoma
Acute Lymphoblastic Lymphoma
 
Gastroenterology
Gastroenterology Gastroenterology
Gastroenterology
 
Carcinoma of unknown primary
Carcinoma of unknown primaryCarcinoma of unknown primary
Carcinoma of unknown primary
 
Bladder cancer
Bladder cancerBladder cancer
Bladder cancer
 
Plasma cell disorders ppt
Plasma cell disorders pptPlasma cell disorders ppt
Plasma cell disorders ppt
 

Dernier

How to Add a many2many Relational Field in Odoo 17
How to Add a many2many Relational Field in Odoo 17How to Add a many2many Relational Field in Odoo 17
How to Add a many2many Relational Field in Odoo 17Celine George
 
Presentation on the Basics of Writing. Writing a Paragraph
Presentation on the Basics of Writing. Writing a ParagraphPresentation on the Basics of Writing. Writing a Paragraph
Presentation on the Basics of Writing. Writing a ParagraphNetziValdelomar1
 
2024.03.23 What do successful readers do - Sandy Millin for PARK.pptx
2024.03.23 What do successful readers do - Sandy Millin for PARK.pptx2024.03.23 What do successful readers do - Sandy Millin for PARK.pptx
2024.03.23 What do successful readers do - Sandy Millin for PARK.pptxSandy Millin
 
DUST OF SNOW_BY ROBERT FROST_EDITED BY_ TANMOY MISHRA
DUST OF SNOW_BY ROBERT FROST_EDITED BY_ TANMOY MISHRADUST OF SNOW_BY ROBERT FROST_EDITED BY_ TANMOY MISHRA
DUST OF SNOW_BY ROBERT FROST_EDITED BY_ TANMOY MISHRATanmoy Mishra
 
Quality Assurance_GOOD LABORATORY PRACTICE
Quality Assurance_GOOD LABORATORY PRACTICEQuality Assurance_GOOD LABORATORY PRACTICE
Quality Assurance_GOOD LABORATORY PRACTICESayali Powar
 
Education and training program in the hospital APR.pptx
Education and training program in the hospital APR.pptxEducation and training program in the hospital APR.pptx
Education and training program in the hospital APR.pptxraviapr7
 
How to Add Existing Field in One2Many Tree View in Odoo 17
How to Add Existing Field in One2Many Tree View in Odoo 17How to Add Existing Field in One2Many Tree View in Odoo 17
How to Add Existing Field in One2Many Tree View in Odoo 17Celine George
 
How to Show Error_Warning Messages in Odoo 17
How to Show Error_Warning Messages in Odoo 17How to Show Error_Warning Messages in Odoo 17
How to Show Error_Warning Messages in Odoo 17Celine George
 
The basics of sentences session 10pptx.pptx
The basics of sentences session 10pptx.pptxThe basics of sentences session 10pptx.pptx
The basics of sentences session 10pptx.pptxheathfieldcps1
 
5 charts on South Africa as a source country for international student recrui...
5 charts on South Africa as a source country for international student recrui...5 charts on South Africa as a source country for international student recrui...
5 charts on South Africa as a source country for international student recrui...CaraSkikne1
 
Patterns of Written Texts Across Disciplines.pptx
Patterns of Written Texts Across Disciplines.pptxPatterns of Written Texts Across Disciplines.pptx
Patterns of Written Texts Across Disciplines.pptxMYDA ANGELICA SUAN
 
AUDIENCE THEORY -- FANDOM -- JENKINS.pptx
AUDIENCE THEORY -- FANDOM -- JENKINS.pptxAUDIENCE THEORY -- FANDOM -- JENKINS.pptx
AUDIENCE THEORY -- FANDOM -- JENKINS.pptxiammrhaywood
 
HED Office Sohayok Exam Question Solution 2023.pdf
HED Office Sohayok Exam Question Solution 2023.pdfHED Office Sohayok Exam Question Solution 2023.pdf
HED Office Sohayok Exam Question Solution 2023.pdfMohonDas
 
Human-AI Co-Creation of Worked Examples for Programming Classes
Human-AI Co-Creation of Worked Examples for Programming ClassesHuman-AI Co-Creation of Worked Examples for Programming Classes
Human-AI Co-Creation of Worked Examples for Programming ClassesMohammad Hassany
 
Prescribed medication order and communication skills.pptx
Prescribed medication order and communication skills.pptxPrescribed medication order and communication skills.pptx
Prescribed medication order and communication skills.pptxraviapr7
 
CAULIFLOWER BREEDING 1 Parmar pptx
CAULIFLOWER BREEDING 1 Parmar pptxCAULIFLOWER BREEDING 1 Parmar pptx
CAULIFLOWER BREEDING 1 Parmar pptxSaurabhParmar42
 
How to Use api.constrains ( ) in Odoo 17
How to Use api.constrains ( ) in Odoo 17How to Use api.constrains ( ) in Odoo 17
How to Use api.constrains ( ) in Odoo 17Celine George
 
CapTechU Doctoral Presentation -March 2024 slides.pptx
CapTechU Doctoral Presentation -March 2024 slides.pptxCapTechU Doctoral Presentation -March 2024 slides.pptx
CapTechU Doctoral Presentation -March 2024 slides.pptxCapitolTechU
 

Dernier (20)

How to Add a many2many Relational Field in Odoo 17
How to Add a many2many Relational Field in Odoo 17How to Add a many2many Relational Field in Odoo 17
How to Add a many2many Relational Field in Odoo 17
 
Presentation on the Basics of Writing. Writing a Paragraph
Presentation on the Basics of Writing. Writing a ParagraphPresentation on the Basics of Writing. Writing a Paragraph
Presentation on the Basics of Writing. Writing a Paragraph
 
2024.03.23 What do successful readers do - Sandy Millin for PARK.pptx
2024.03.23 What do successful readers do - Sandy Millin for PARK.pptx2024.03.23 What do successful readers do - Sandy Millin for PARK.pptx
2024.03.23 What do successful readers do - Sandy Millin for PARK.pptx
 
DUST OF SNOW_BY ROBERT FROST_EDITED BY_ TANMOY MISHRA
DUST OF SNOW_BY ROBERT FROST_EDITED BY_ TANMOY MISHRADUST OF SNOW_BY ROBERT FROST_EDITED BY_ TANMOY MISHRA
DUST OF SNOW_BY ROBERT FROST_EDITED BY_ TANMOY MISHRA
 
Quality Assurance_GOOD LABORATORY PRACTICE
Quality Assurance_GOOD LABORATORY PRACTICEQuality Assurance_GOOD LABORATORY PRACTICE
Quality Assurance_GOOD LABORATORY PRACTICE
 
Education and training program in the hospital APR.pptx
Education and training program in the hospital APR.pptxEducation and training program in the hospital APR.pptx
Education and training program in the hospital APR.pptx
 
How to Add Existing Field in One2Many Tree View in Odoo 17
How to Add Existing Field in One2Many Tree View in Odoo 17How to Add Existing Field in One2Many Tree View in Odoo 17
How to Add Existing Field in One2Many Tree View in Odoo 17
 
How to Show Error_Warning Messages in Odoo 17
How to Show Error_Warning Messages in Odoo 17How to Show Error_Warning Messages in Odoo 17
How to Show Error_Warning Messages in Odoo 17
 
The basics of sentences session 10pptx.pptx
The basics of sentences session 10pptx.pptxThe basics of sentences session 10pptx.pptx
The basics of sentences session 10pptx.pptx
 
Prelims of Kant get Marx 2.0: a general politics quiz
Prelims of Kant get Marx 2.0: a general politics quizPrelims of Kant get Marx 2.0: a general politics quiz
Prelims of Kant get Marx 2.0: a general politics quiz
 
5 charts on South Africa as a source country for international student recrui...
5 charts on South Africa as a source country for international student recrui...5 charts on South Africa as a source country for international student recrui...
5 charts on South Africa as a source country for international student recrui...
 
Patterns of Written Texts Across Disciplines.pptx
Patterns of Written Texts Across Disciplines.pptxPatterns of Written Texts Across Disciplines.pptx
Patterns of Written Texts Across Disciplines.pptx
 
AUDIENCE THEORY -- FANDOM -- JENKINS.pptx
AUDIENCE THEORY -- FANDOM -- JENKINS.pptxAUDIENCE THEORY -- FANDOM -- JENKINS.pptx
AUDIENCE THEORY -- FANDOM -- JENKINS.pptx
 
HED Office Sohayok Exam Question Solution 2023.pdf
HED Office Sohayok Exam Question Solution 2023.pdfHED Office Sohayok Exam Question Solution 2023.pdf
HED Office Sohayok Exam Question Solution 2023.pdf
 
Human-AI Co-Creation of Worked Examples for Programming Classes
Human-AI Co-Creation of Worked Examples for Programming ClassesHuman-AI Co-Creation of Worked Examples for Programming Classes
Human-AI Co-Creation of Worked Examples for Programming Classes
 
Prescribed medication order and communication skills.pptx
Prescribed medication order and communication skills.pptxPrescribed medication order and communication skills.pptx
Prescribed medication order and communication skills.pptx
 
CAULIFLOWER BREEDING 1 Parmar pptx
CAULIFLOWER BREEDING 1 Parmar pptxCAULIFLOWER BREEDING 1 Parmar pptx
CAULIFLOWER BREEDING 1 Parmar pptx
 
Finals of Kant get Marx 2.0 : a general politics quiz
Finals of Kant get Marx 2.0 : a general politics quizFinals of Kant get Marx 2.0 : a general politics quiz
Finals of Kant get Marx 2.0 : a general politics quiz
 
How to Use api.constrains ( ) in Odoo 17
How to Use api.constrains ( ) in Odoo 17How to Use api.constrains ( ) in Odoo 17
How to Use api.constrains ( ) in Odoo 17
 
CapTechU Doctoral Presentation -March 2024 slides.pptx
CapTechU Doctoral Presentation -March 2024 slides.pptxCapTechU Doctoral Presentation -March 2024 slides.pptx
CapTechU Doctoral Presentation -March 2024 slides.pptx
 

Endocrinology archer step3 lectures

  • 1. ENDOCRINOLOGY Archer Online USMLE Reviews www.ArcherReview.com All rights reserved Archer Slides are intended for use with Archer USMLE step 3 video lectures. Hence, most slides are very brief summaries of the concepts which will be addressed in a detailed way with focus on High-yield concepts in the Video lectures.
  • 2. Thyroid Disorders Thyrotoxicosis Subacute Thyroiditis Thyroid Function Tests Hashimatos Thyroiditis Hypothyroidism Thyrotoxic periodic paralysis Thyroid Nodule and Approach Thyroid Cancer
  • 3. Thyroid Diagnostic Studies  Thyroid Function Tests : TSH, Total T4, Free T4, T3 levels.  Thyroid Binding Globulin  Thyroid Uptake Scan – Radio active iodine uptake scan ( RAIU scan)  Antibodies : Thyroid stimulating immunoglobulins, Anti-microsomal antibodies, Anti-thyroid peroxidase antibodies  Thyroid ultrasound  Thyroid Biopsy, FNAC
  • 4. TSH Obtain screening TSH in patients with conditions that may be explained by or worsened by hyperthyroidism :  Unexplained weight loss  Anxiety or sleep disturbance  Tachycardia, including supraventricular tachycardia and new onset atrial fibrillation  Osteoporosis ( always suspect and rule out hyperthyroidism especially when osteoporosis is worsening despite adequate therapy in the elderly)
  • 5. TSH Obtain screening TSH in patients with conditions that may be explained by or worsened by hypothyroidism :  History of thyroid disease  Autoimmune disease (R/O co-existent hashimatos)  Unexplained depression  Cognitive dysfunction ( dementia, delirium)  Hypercholesterolemia ( can be secondary to hyopthyroidism or can be worsended by it)  Unexplained constipation Screen in early pregnancy
  • 6. TSH  Preferred Screening test for suspected HYPO/HYPER Thyroidism  Preferred Follow-up test for patients receiving therapy for either hypo or hyperthyrodism or pts who had thyroid cancer therapy  If TSH High  possible Primary hypothyroidism or rarely, secondary hyperthyroidism  If TSH low  possible Primary hyperthyroidism or rarely, secondary hypothyroidism  If TSH low in pt on treatment for hypothyroidism  reduce dose of levothyroxine. ( usually in increments of 25mcg each time)  If TSH high in a pt on treatment for hypothyroidism  Increase dose of levothyroxine. ( usually in increments of 25mcg each time)  If TSH low in pt on treatment for hyperthyroidism  Increase dose of Propylthiouracil/ Methimazole ( or may indicate inadequate therapy)  If TSH high in a pt on treatment for hyperthyroidism  Reduce dose of Propylthiouracil/ Methimazole ( Indicates excess antithyroid therapy.  If Radioiodine therapy was chosen, then aim is to make the pt Hypothyroid in order to achieve good cure rates in Graves thyrotoxicosis  in that case, you need to start levothyroxine post radioiodine therapy or post surgically)
  • 7. TFTs – Further Approach Obtain Total and Free T4 levels, ONCE TSH comes abnormal
  • 8. TFTs – Further Approach  If TSH high but Free T4 low  primary hypothyroidism  Usually Hashimatos thyroiditis.  In countries where Iodine deficiency is a problem, this could be due to endemic goiter. (urine iodine low)  Iatrogenic Hypothyroidism due to Thyroidectomy or Radioiodine therapy  Drug induced  Amiodarone, Lithium, Iodine {excess iodine immediately inhibits both new hormone synthesis (by blocking organification, known as the Wolf-Chaikoff effect) and the release of thyroid hormone. It also decreases gland size and vascularity. This effect is transient, however, lasting approximately 1-3 weeks}, Interferon, Thionamides  Late phase of Post-partum thyroiditis, Subacute thyroiditis, silent thyroiditis
  • 9. TFTs – Further Approach  If TSH High but Free T4 also high  secondary Hyperthyroidism ( pituitary adenomas) - rare  Get a Pituitary MRI
  • 10. TFTs – Further Approach  If TSH high but Free T4 normal  Subclinical Hypothyroidism  Subclinical Hypothyroidism :  Treatment recommended only :  If clinical symptoms are present  If hyperlipidemia (high LDL)  If patient has goiter or  If TSH > 10mU/l , For TSH between 5 to 10  obtain TPO antibodies and consider treatment if TPO antibodies high)  In cases of subclinical hypothyroidism, presence of anti-TPO antibodies predicts progression to overt Hypothyroidism ( Hashimato’s)
  • 11. TFTs – Further Approach  If TSH low but Free T4 high  primary hyperthyroidism  Usually Graves Thyrotoxicosis ( Thyroid scan – homogenous, diffuse uptake)  Toxic Multinodular Goiter ( Thyroid scan – Heterogenous)  Toxic Solitary Adenoma  Early phase of Subacute Thyroiditis/ Silent Thyroiditis ( No Uptake on Thyroid scan)  Post-partum Thyroiditis ( painless, may be followed by hypothyroid phase)  Iatrogenic Hyperthyroidism due to excess levothyroxine therapy in a hypothyroid patient  Surreptious intake of Thyroid hormones  Struma Ovarii ( Increased pelvic uptake on whole body iodine scan)  Gestational Thyrotoxicosis ( Increased HCG is responsible, Nuclear imaging contraindicated here)  Amiodarone induced ( two types of thyrotoxicosis)
  • 12. TFTs – Further Approach  If TSH low but Free T4 also low  secondary Hypothyroidism  ( pituitary adenomas causing compression, craniopharyngioma)  clues are headaches, visual field deficits, ophthalmoplegia), pituitary/ hypothalamic surgery, pituitary/ hypothalamic infection/ infarction  Next step, get a Pituitary MRI
  • 13. TFTs – Futher Approach  If TSH low but Free T4 normal  get T3 level.  If T3 high  T3 toxicosis.  If T3 normal  Subclinical Hyperthyroidism  Get an RAIU scan to see if this is Graves or a Toxic adenoma .  Subclinical hyperthyroidism can be just followed if no clinical symptoms and if there is no significant reason to treat.  Treatment of subclinical hyperthyroidism is recommended  in cases of osteoporosis ( Thyroxine increases bone resorption)  In atrial fibrillation  in elderly due to risk of atrial fibrillation and osteoporosis. (If Graves, thionamide or I131. If Toxic adenoma, I131 is preferred.)
  • 14. TFTs  Remember any combinations of TSH/Free T4 can occur in severe nonthyroidal illness depending on the phase of the illness ( critical phase, recovery phase)  Euthyroid sick syndrome . Some combinations  low TSH, Normal free T4 ( may look like subclinical hyperthyroidism)  High TSH, Normal free T4 ( recovery phase, may look like subclinical hypothyroidism)  Your clue to answer questions in such situations is the history recent or ongoing critical illness.
  • 15. Thyrotoxicosis Important topics: Graves disease Subacute thyroiditis Post partum thyroiditis Amiodarone induced
  • 17. Classification of Thyrotoxicosis based on RAIU Scan High RAIU Low RAIU Graves' disease Subacute thyroiditis Toxic multinodular goiter* Painless thyroiditis (includes postpartum thyroiditis) Toxic adenoma* Chronic thyroiditis with transient thyrotoxicosis HCG-mediated (choriocarcinoma) Iatrogenic/surreptitious thyroid hormone TSH-secreting pituitary tumor Metastatic functional follicular thyroid cancer (after thyroidectomy) Amiodarone-induced thyroiditis Struma ovarii (uptake in pelvis)
  • 18. Grave’s Disease  Features : Diffuse Goiter, Ophthalmopathy ( proptosis, extraocular muscle paralysis, peri- orbital edema are specific. Lid-lag and stare can occur with any hyperthyroidism ) and Dermopathy (pre-tibial myxedema) .  Diagnosis : clinical features of hyperthyroidism ( palpitations, sweating, diarrhea, anxiety, tremor, hairloss, weightloss, menorrhagia) + diffuse goiter, thyroid bruit/thrill can be heard due to hyperdynamic circulation and increased thyroid vascularity.  Pain usually absent in thyroid ( PAIN may be present in subacute thyroiditis)  Always r/o subacute thyroiditis in differential  Obtain RAIU scan  Graves shows homogeneous uptake where as no uptake in subacute thyroiditis/ Surreptious intake of LT4. { Measure serum thyroglobulin and ESR to distinguish subacute thyroiditis (elevated thyroglobulin and ESR) from surreptitious thyroid hormone ingestion (suppressed thyroglobulin, normal ESR)}. Painless thyroiditis has elevated thyroglobulin but normal ESR.  TSH low, Free T4 high.  Antibodies present  Thyroid stimulating antibodies ( TSIgs) are high and are responsible for goiter and hyperthyroidism. Anti-TSH antibodies, Anti-TPO antibodies.  Can be associated with other autoimmune diseases  Vitilgo, pernicious anemia
  • 19. Treatment Options  Treatment options in Hyperthyroidism :  Medications ( Antithyroid drugs - PTU, Thionamides i.e; methimizole ) , B-Blockers ( propranolol, atenolol) for symptoms. ( propranolol preferred because it can block T4 to T3 conversion in high doses)  Raioactive iodine therapy (131 I)  Surgery ( Thyroidectomy)  Consider the following when selecting treatment for thyrotoxicosis:  Patient preference  Patient's age and comorbidities  Severity of thyrotoxicosis  Goiter size  Presence of ophthalmopathy (in Graves' disease patients)
  • 20. Treatment for Thyrotoxicosis Due to Overproduction of Thyroid Hormone Modality Advantages Disadvantages Antithyroid drug therapy Least invasive Least costly Lower risk of permanent hypothyroidism Potential beneficial immunomodulatory effects Adverse drug effects Low permanent remission rate (approximately 33%-50%) Radioiodine Moderately fast reduction in thyroid hormone levels Less invasive than surgery Permanent hypothyroidism likely Requires delay in pregnancy (6-12 months) and breastfeeding May precipitate new or worsened ophthalmopathy Slight risk of thyroid storm after treatment ( in severe Hyperthyroidism, use pre- treatment) Thyroidectomy Rapid reduction in thyroid hormone levels Allows concurrent removal of suspicious nodules (if present) Allows reduction in size of large goiters Most invasive Most costly Permanent scar Permanent hypothyroidism likely Potential injury to parathyroids and recurrent laryngeal nerve
  • 21. Anti-thyroid drug therapy  Propyl thiouracil :  Typical starting dose 100mg tid  Will need to discontinue 2 weeks prior to Radioiodine therapy because it can lead to failure of the therapy as it inhibits roadioiodine uptake by thyroid. ( REMEMBER METHIMIZOLE WILL NOT MUCH AFFECT RADIOIODINE UPTAKE – So, it is the preferred therapy to start if I131 therapy is planned but stop it 3 days prior to I131)  LESS POTENT than Methimizole.  Preferred drug in severe Hyperthyroidism because it can block T4 to T3 Conversion ( this effect not seen with methimazole)  Side effects : Rash, Agranulocytosis, Hepatocellular necrosis  Safe in pregnancy ( as it highly binds to plasma protein and very little crosses placenta. There is NO risk of Aplasia Cutis). However, Fetal hypothyroidism can occur in high doses due to transplacetal passage.  MONITOR LFTS AND CBC  If pts have sorethroat/ infection etc suggesting possibility of agaranulocytosis d/c PTU and get a blood count
  • 22. Anti-thyroid drug therapy  Methimizole :  More potent than PTU  Typical starting dose is 30mg/d – preferred drug for most patients because single daily dose (except for patients with allergy to methimazole, who are pregnant, or have severe thyrotoxicosis or thyroid storm)  Contraindicated in pregnancy due to risk of Aplasia cutis and higher transplacental spread than PTU.  Not preferred in breast feeding ( PTU is preferred in breast feeding)  Side effects : Rash, Agranulocytosis, cholestatic jaundice  Good for pre-treatment prior to I-131 therapy. ( pre-treatment is used to normalize thyroid function before the administration of radioiodine and to attenuate potential exacerbations following ablative radioiodine therapy.  recommended for the elderly and those with underlying cardiac disease, who may be more vulnerable to worsening thyrotoxicosis during I 131 therapy.). Stop pre-treatment with methimazole 3 days prior to I131 to prevent treatment failure. Restart 3 days after giving I131 ( I131 takes 12 to 15 weeks to correct hyperthyroidism – monitor free T4 after I131 Therapy)  PTU is preferred over methimizole in severe hyperthyroidism.  IF USING ANTITHYROID THERAPY as sole primary therapy use for at least 12-18 months before tapering.  When started, Thionamides can correct hyperthyroidism in about 4 to 8 weeks ( response expected in 4 to 8 wks)
  • 23. Antithyroid drug therapy  For Grave’s disease patient getting only anti thyroid drug therapy, factors favoring remission are : - Small goiter - Low T3/T4 ratio - Mild thyrotoxicosis - Negative TSH-receptor antibody titers after treatment - No prior relapse  So, antithyroid drug therapy can be a preferred modaility in these scenarios.
  • 24. Aplasia Cutis  Absence of skin in the newborn  Can occur with Methimizole use during pregnancy.  PIC : Aplasia Cutis of scalp
  • 25. Radioiodine therapy  Select a dose of radioiodine based on the size of the patient's goiter and the result of the RAIU test.  Do not use radioactive iodine in patients who are pregnant (absolute contraindication) or breastfeeding.  Pregnancy should be delayed four to six months after radioiodine therapy.  Radioiodine is better avoided in patients with Graves' ophthalmopathy. ( may cause worsening of ophthalmopathy)  After Radioiodine therapy  mother should stay away from children at least for 7 days ( Mother need to Arrange for a care taker for baby).  A I131 recepient must avoid contact with pregnant women, avoid sexual contact for one week after the therapy
  • 26. Radioiodine Therapy – Follow Up  Hypothyroidism is inevitable in most cases. The risk of Hypothyroidism depends on the dose of I131.  About 2/3 of the patients develop hypothyroidism in the first year after I131. Even if they do not develop hypothyroidism in the first year, the TSH should be followed every 6 months forever after I131 therapy since the risk of developing hypothyroidism persists for lifetime  Get free T4 in the early period after I131 therapy not TSH. TSH can be misleading in that it can remain low for about 4 to 6 weeks even when patient’s free T4 returns to normal and patient is clinically euthyroid.  I131 typically takes 12 to 15 weeks to correct hyperthyroidism. So, thionamide has to be restarted 3 days after I131 and Free T4 and TSH carefully monitored. Once there is evidence of euthyroid state, taper and stop thionamide. Follow TSH, if hypothyroidism sets in  start Levothyroxine)  Start Levothyroxine after getting TSH level and Free T4 if the levels are consistent with hypothyroidism .
  • 27. Surgery  Especially preferred in :  Goiter causing obstructive symptoms  Pregnant women allergic to PTU or develop agranulocytosis on PTU ( Radio iodine is not an option in pregnancy)  Intolerance to thionamides; refusal to take antithyroid drugs or radioactive iodine  Recurrence after a trial of thionamide therapy for Graves’ disease  Large MULTI-NODULAR goiter  Suspicion of malignancy.  Patients should be made euthyroid with thionamide therapy prior to elective surgery. Once a euthyroid state is achieved, oral iodine is given 7 to 10 days preoperatively to reduce the vascularity of the gland  then total/ subtotal thyroidectomy.  Complications : Hypothyroidism inevitable, Recurrent laryngeal nerve palsy (1%) , Permanent hypoparathyroidism ( 1% risk) and 1% risk of recurrent hyperthyroidism  Admit pts to ICU for 24 hrs s/p surgery  Monitor calcium level q6hrs in patents s/p thyroid surgery and replete calcium aggressively if <7mg%
  • 28. Follow-up  In pts on long term antithyroid drug therapy  Monitor for adverse effects of antithyroid drugs such as rash, hepatic dysfunction, and agranulocytosis.  Repeat thyroid function tests every 3 to 6 months for the first year and then every 6 to 12 months after attaining normal FT4 and TSH levels.  Follow serum FT4 and TSH levels at 4-week intervals to ensure adequate replacement therapy ( for hypothyroidism) immediately after radioiodine therapy.
  • 29. Q1  A 58-year-old woman has anxiety, tremors, excessive sweating, palpitations, and insomnia of approximately 1 month’s duration. Her medical history is unremarkable. She has had no recent pregnancies or miscarriages. She has a modest, nontender goiter and no exophthalmos. She takes no medications and has had no recent radiologic procedures. The  24-hour radioactive iodine uptake is 10% (normal, 20% to 35%).  Laboratory Studies  Erythrocyte sedimentation rate 8 mm/h  Free thyroxine 3.5 ng/dL  Thyroid-stimulating hormone < 0.01 μU/mL  Thyroglobulin 45 ng/mL (normal, 2 to 20 ng/mL) ( do not confuse thyroglobulin with thyroid binding globulin – thyroglobulin is the one present in the thyroid follicles and is released out and elevated whenever there is “thyroidITIS”  Antithyroperoxidase 26 (normal, <2 μU/mL) What is the most likely diagnosis?  ( A ) Struma ovarii  ( B ) Recent imaging study with an iodinated contrast  ( C ) Subacute thyroiditis  ( D ) Surreptitious use of thyroid hormones  ( E ) Painless/silent thyroiditis  (F) Use of Amiodarone
  • 30. Q2  A 58-year-old woman presents to your office with increasing fatigue, memory loss, and depression for the past one year. Her history is significant for hypertension and she takes enalapril for it. Her family history reveals a brother who takes a medication for his thyroid. On physical examination, she is 168 cm (66 in) tall and weighs 73 kg (162 lb). Blood pressure is 152/88 mm Hg and heart rate is 86/min. HEENT exam is nornal without any thyroid enlargement.  Laboratory Studies  Hematocrit 46%  Plasma glucose 82 mg/dL  Total cholesterol 255 mg/dL  High-density lipoprotein cholesterol 49 mg/dL  Low-density lipoprotein cholesterol 187 mg/dL  Thyroid-stimulating hormone 12.2 μU/mL  A free thyroxine level of 0.9 ng/dL.  Which of the following statements is correct about levothyroxine replacement therapy for this patient?  ( A ) Her LDL will decrease.  ( B ) There will be no improvement in her symptoms.  ( C ) She is likely to lose weight.  ( D ) She will be at an increased risk of atrial fibrillation  ( E ) she will have an increased risk of osteoporosis
  • 31. ANS.A  This patient has subclinical hypothyroidism. Rx is recommended if serum TSH levels greater than 10 μU/mL. Also, Rx is recommended in patients having symptoms and effects of hypothyroidism like hyperlipidemia In such cases, levothyroxine therapy reduces LDL cholesterol levels and improves symptoms.
  • 32. Q3 A 50 y/o HIV +ve man with a CD4 count of 200 prrsents with complaints of fatigue, weightloss and dry cough. Cough has been for past 2 days and has high grade fever. No chest pain but he is visibly short of breath. His ABGs revealed hypoxemic respiratory failure with a po2 of 45. He is started on Bactrim IV and steroids.  Laboratory Studies reveal  Free thyroxine 0.9 ng/dL  Triiodothyronine 22 ng/dL  Thyroid-stimulating hormone 0.3 μU/mL  Which of the following statements about this patient’s condition is true?  ( A ) Perform a radioactive iodine uptake test  ( B ) Using T3 treatment will improve his condition and prognosis.  ( C ) The lab abnormalities are due to decreased peripheral conversion of thyroxine to triiodothyronine  ( D ) Serum thyroid binding globulin is increased  ( E ) Pituitary MRI should be obtained
  • 34. Euthyroid Sick Syndrome  Euthyroid sick syndrome is seen in acutely and critically ill patients.  Occurs due to alterations in the levels of circulating thyroid hormones that can occur in severe nonthyroidal illnesses.  Features: low T3, high reverse t3 normal T4, and low TSH levels. TSH starts elevating to hypothyroid levels during “recovery” phase of non thyroidal illness and returns to normal with complete recovery. But many combinations can occur
  • 35. Q4  A 32-year-old woman is evaluated for recent onset of fatigue, palpitations, profuse sweating, and emotional lability. She gave birth to her second child 8 weeks ago and is not breast feeding. On physical examination, her pulse rate is 100/min, and she has mild lid lag, a fine hand tremor, and a slightly enlarged, nontender thyroid gland. Radioactive iodine uptake is less than 1% at 4 and 24 hours (normal, 20% to 35%). Laboratory test results include a serum thyroid-stimulating hormone level of less than 0.03 μU/mL and a free thyroxine level of 3.8 ng/dL.  Which of the following is the optimal treatment for this patient?  ( A ) Antithyroid drugs  ( B ) Radioactive iodine  ( C ) Thyroidectomy  ( D ) Prednisone  ( E ) ß-blockers
  • 36. Key Point  Women presenting with thyrotoxicosis are treated with beta blockers to decrease palpitations tremors..  Antithyroid medications are not used in the thyrotoxic phase as since thyroid is not overactive.  Post partum thyroiditis may be followed by a hypothyroid phase ( due to destruction – which may start 4-8 months post-partum and may last 9-12 months) - treated with thyroid hormone replacement  which should be later tapered off  It is always important to monitor and taper off thyroid hormone after postpartum thyroiditis, since 80% of patients will regain normal thyroid function and not require chronic therapy.
  • 37. IMP! – AIT  This condition occurs in up to 10% of patients who use amiodarone, which has a very high iodine content. Two subtypes occur: type 1 AIT is caused by iodine overload and occurs primarily in patients with underlying goiters; type 2 AIT is caused by drug-induced thyroid follicular damage (thyroiditis).  Both types are associated with a low 24-hour radioactive iodine uptake. No tests reliably distinguish between the two subtypes, although an underlying goiter and detectable radioactive iodine uptake are more common in type 1  AIT. Treatment of type 1 AIT consists of administering thionamides, with or without potassium perchlorate  Type 2 AIT may respond to corticosteroid therapy.  Patients who do not respond to this treatment may require plasmapheresis, dialysis, or thyroidectomy.
  • 38.  A 70-year-old man is evaluated because of a 1-year history of progressive weakness, weight loss, and hand tremors. For nearly 3 years, he has been treated with amiodarone for paroxysmal atrial flutter. He has no goiter and no history of thyroid disease. Thyroid scan shows scant, patchy tracer uptake. The 24-hour radioactive iodine uptake is 2.7% at 6 hours and 4.1% at 24 hours (normal, 20% to 35%). The serum thyroid- stimulating hormone level is <0.01 μU/mL and the serum free thyroxine level is 3.8 ng/dL.  Which of the following statements is true about the effects of amiodarone on this patient’s thyroid function?  ( A ) Amiodarone can cause thyrotoxicosis by producing iodine overload or thyroiditis.  ( B ) Amiodarone blocks peripheral thyroid hormone receptors.  ( C ) Amiodarone increases peripheral conversion of thyroxine to triiodothyronine.  ( D ) Amiodarone directly suppresses pituitary secretion of thyroid-stimulating hormone.  ( E ) Amiodarone increases serum thyroid hormone protein binding.
  • 39.  Two subtypes of amiodarone-induced thyrotoxicosis occur: type 1 is caused by iodine overload and occurs primarily in patients with underlying goiters; type 2 is caused by drug- induced thyroid follicular damage (thyroiditis).
  • 41. Hashimato’S Thyroiditis  Autoimmune thyroiditis associated with lymphocytic infiltration of thyroid gland.  causes gland disruption, initially may lead to hashitoxicosis and then hypothyroidism  Diagnosis : HIGH TSH, LOW FREE T4 and Elevated TPO antibodies/ anti microsomal antibodies.  TSIgs may be present too  Diffuse Goiter on physical exam.  Association with other autoimmune diseases must be kept in mind  pernicious anemia, addisons disease  Can be associated with B cell lymphomas
  • 42. Treating Hypothyroidism  Levothyroxine (LT4) drug of choice.  In young patients  start initial full replacement dose calculated as 1.6 µg/kg/d. ( 70kg man – 112mcg/d)  In obese patients  calculate the initial dose using ideal body weight  of course , you can follow-up serum TSH 6-8 weeks later and titrate the dose.  In elderly patients > 60 yrs of age  start at dose 25 to 50 mcg and titrate every 6-8 wks untill TSH comes to desired range.  In those with KNOWN CAD/ hx of CHF and arrhythmias  Start at 12.5 mcg to 25 mcg dose and then titrate every 6-8 wks untill TSH comes to desired range.
  • 43.  Ideal Body Weight (men) = 50 + 2.3 ( Height(in) - 60 )  Ideal Body Weight (women) = 45.5 + 2.3 ( Height(in) - 60 )
  • 44. Follow up – Treatment adequacy  TSH is used to follow up treatment adequacy  Levothyroxine has a t 1/2 of 7days. It takes a drug four to five t1/2s to reach steady state  which means DO NOT INCREASE LT4 dose prior to 4 to 6 weeks if pt is not symptomatic.  TSH level should be obtained at 6 – 8 weeks after starting therapy  because it takes this time for raised to TSH to normalize and also this is the time at which you can properly assess adequacy of LT4 therapy because LT4 reaches steady state around this time.
  • 45. Follow up – Treatment adequacy  Use TSH levels as the guide to thyroid hormone dosage requirements.  Once treatment is started, monitor serum TSH levels every 6 to 8 weeks and adjust the LT4 dose until the TSH value is in the desired range.  Once TSH reaches desired range, recheck the TSH level 3 to 6 months later, and then if normal recheck annually.  If the TSH value is above the normal range, increase the LT4 dose by 12.5 to 25 µg/d and recheck in 6 to 8 weeks.  If the TSH value is low, decrease the LT4 dose by 12.5 to 25 µg/d and recheck in 6 to 8 weeks
  • 46. Recognize Interactions!  Counsel patients to take levothyroxine at the same time each day and to avoid taking it within 4 hours of iron tablets, calcium supplements, antacids, and fiber supplements ( can reduce LT4 absorption)
  • 47. Lithium Induced  Li Can cause hypothroidism  Rx  do not discontinue lithium, Start levothyroxine and continue lithium therapy.
  • 48. Myxedema Coma  Severe hypothyroidism associated with altered mental status/ delirium/ coma.  Myxedema coma  usually found in elderly patients who have untreated or inadequately treated hypothyroidism and then develop a precipitating event. ( precipitating event for myxedema coma in a pt with untreated/inadequately treated hypothyroidism can be SEPSIS, Surgery, Trauma, GI bleeding, Stroke, MI, CHF, Sedative use)  So, always evaluate the pt for precipitating factor also  treat both myxedema coma as well as precipitant.
  • 49. Myxedema coma  Admit to ICU for careful monitoring and appropriate treatment.  Get Endocrine consult always.  Obtain blood cultures, CT head, cardiac enzymes, EKG, CHEM 18, CBC R/o preciptating factors  TSH, FREE T4  RX :  Vital support : intubation if prolonged coma for airway protection, IV fluids, hypothermia blanket.  Levothyroxine intravenosly (500 µg, then 50-100 µg/d for myxedema coma, for npo pts 80% of oral dose ) + hydrocortosine 100 q8hrs x 2days to address possible decreased adrenal reserve in myxedema.  Also can use combination of LT4+LT3 for rapid T4 and T3 repletion in myxedema coma  Look for and treat any precipitating factor
  • 51. Approach – Palpable Thyroid Nodule  If thyroid nodule palpable  Get TSH First.  If High TSH – suggests cold nodule/ Hashimatos  Get FNAC ( some recommend ultrasound as next step here because hashimatos may have benign nodularity that regress with therapy and ultrasound will help to see if there are suspicious features. If U/S suspicious, then FNAC is recommended. This may be optimal approach because hurthle cells of hashimatos may cause false positives on cytology if the FNAC is obtained from such benign nodule). Treat with levothyroxine if overt hypothyroidism or subclinical hypothyroidism that warrants treatment.  If TSH normal – suggests cold nodule - get FNAC next  If TSH low - suggests Hot nodule ( toxic adenoma) but not confirmative (What if there is GRAVES in the surrounding tissue and this is a cold nodule?)  so, next step get RAIU scan  if COLD nodule, get FNAC. If RAIU scan shows Hot nodule treat with I131 ( if there is overt hyperthyroidism from this toxic adenoma) or just observation.  Cold nodules are more likely to be malignant when compared to hot nodules ( hot/ functioning nodule virtually rules out malignancy)  A negative FNAC does not rule out cancer completely. So, if clinical suspicion for cancer is high  consider surgical excision of the entire nodule. ( especially, when histology revealed follicular adenoma – remember, a negative FNAC from one area may not rule out possibility of follicular carcinoma in follicular adenoma )
  • 52. Approach – Thyroid Incidentalomas  Thyroid Incidentalomas – These are those nodules ( not the palpable ones) detected on ultrasound such as ultrasound done for other palpable thyroid abnormalities, during carotid artery imaging or ultrasound done for hyperparathyroidism).  FNAC indicated if  Nodule > 10 mm in diameter.  On ultrasound, if nodule has suspicious features of malignancy  hypoechoic, microcalcifications, irregular shape, blurred margin or increased vascularity  If there are risk factors for thyroid cancer ( family history, childhood neck irradiation)
  • 53. After FNAC - Further Approach  If FNAC is benign  Cystic nodule – use percutaneous Ethanol injection to get rid of nodule  Solid Nodule – use suppressive therapy with levothyroxine if there are no contraindications ( Heart disease, old age). Suppressive therapy with LT4 aims to reduce TSH and there by, regress the nodule  If FNAC is malignant  SURGERY  If FNAC is follicular adenoma (benign)  get a thyroid scan if not already done  If this is hot/functioning nodule, it rules out malignancy – so, follow-up. If this is a cold nodule, a negative FNAC from one area may not rule out possibility of follicular carcinoma in follicular adenoma – so, do Surgical excision of the entire nodule (sub-total thyroidectomy) in those cases.  If FNAC is non-diagnostic  repeat FNAC
  • 55. Thyroid Cancer - Prevalence Prevalence of thyroid cancer is higher in the following groups and hence, high clinical suspicion should be maintained if you find a thyroid nodule in these groups ( i.e; warrants further evaluation of the nodule):  Children  Adults less than 30 yrs or above 60 yrs of age  History of head and neck radiation (eg: Hodgkins )  Family history of thyroid cancer
  • 56. Radiation Exposure – Follow up for detecting thyroid cancer  Guidelines are not clear.  The incidence of thyroid cancer after radiation exposure is high for about four decades after exposure and then it declines.  If there is a history of significant radiation exposure such as ( therapeutic external beam radiation therapy in children, survivors of Japan atomic bombing, exposure during chernobyl nuclear incident), physical examination by thyroid palpation and ultrasound should be obtained every three years to detect any nodules.  If there is a palpable nodule in these patients, ultrasound must still be obtained to detect any non palpable nodules ( because multiple nodules are a common finding in radiation exposed patients)  All palpable nodules and suspicious nodules on ultrasound, must undergo FNAC.
  • 58. Pheochromocytoma Consider evaluating for Pheochromocytoma in patients with:  Refractory hypertension  Hypertension, accompanied by hyperadrenergic spells with:  Nonexertional palpitations  Diaphoresis  Headache  Tremor  Pallor  Family history of familial pheochromocytoma  A genetic syndrome that increases the risk of pheochromocytoma, such as:  Multiple endocrine neoplasia type 2  von Hippel-Lindau disease - RCC  Neurofibromatosis type 1  History of gastric stromal tumor or pulmonary chondromas (Carney triad)  An incidentally discovered adrenal mass ( Adrenal Incidentalomas)
  • 59. Pheochromocytoma  Episodic Hypertension is charecterestic but ORTHOSTATIC HYPOTENSION can be seen.  Pheochromocytomas are 10% tumors  10% malignant, 10% bilateral and 10% familial.
  • 60. Pheochromocytoma - Diagnosis  Best diagnostic test and high specificity (99%)  24 hr Urinary Metanephrines ( metanephrines or normetanephrines above the upper limit of normal in a patient not taking an interfering medication and not physically stressed is consistent with pheochromocytoma )  Plasma Metanephrines can be used if 24 hr unrine collection is not possible –specificity (88%) lower than 24 hr urine metanephrines  Obtain CT Abdomen to r/o Adrenal mass only if biochemical tests are positive. Very sensitive (96%) for pheochromocytoma ( adrenal mass) but low specificity due to high prevalence of adrenal incidentalomas.  MIBG scan  Very specific for pheochromocytoma. Sensitivity lower than CT scan. (Sensitivity only 80% and specificity 100%) MIBG (Metaiodobenzylguanidine) scan is the best test if looking for extra adrenal pheochromocytomas  Other tests – Plasma catecholamines, Urine VMA
  • 61. False + ves Medications and activities that may increase measured levels of catecholamines and metabolites ( False +ves for Metanephrines/ VMA)  Tricyclic antidepressants  Labetalol  Levodopa  Drugs containing catecholamines (e.g., decongestants)  Amphetamines  Buspirone (and most psychoactive agents)  Sotalol  Withdrawal from clonidine hydrochloride and other drugs  Ethanol  Acetaminophen and phenoxybenzamine (fractionated plasma metanephrines)  Physical stress (e.g., stroke, obstructive sleep apnea)
  • 62. Treatment  Acute Hypertensive crises  use IV Phentolamine, Sodium nitroprusside or Nicardipine.  Definitive treatment is SURGERY. Send for histopath to r/o malignancy.  REMEMBER TO USE BOTH ALPHA AND BETA ADRENERGIC BLOCKING AGENTS PRIOR TO SURGERY – to prevent hypertensive crises during surgery. ( use phenoxybenzamine + atenolol)  Do not use non selective beta blocker without alpha blocker ( eg: propranolol alone)  Understand that relatively short-acting, selective α1- adrenergic receptor blockers (e.g., prazosin, terazosin, doxazosin) may be inadequate for preoperative drug preparation  so phenoxybenzamine preferred for pre- op therapy as it is long acting.
  • 64. Hypercalcemia  Etiology  Clinical features : bones, moans, stones, groans  Investigations: Ca, Phos, EKG, PTH, 24 hr Urinary calcium excretion or spot urine calcium/creatinine ratio ( R/o familial hypocalciuric hypercalcemia)  Management:  Criteria for surgery in primary hyperparathyroidism  Sestamibi scan only if surgery is planned/indicated  Hypercalcemic crisis management – ivf + lasix after volume repletion only  Indications for corticosteroids : are useful for treating hypercalcemia caused by vitamin D toxicity, certain malignancies (eg, multiple myeloma, lymphoma), sarcoidosis, and other granulomatous diseases  Cinacalcet (Sensipar) -- Directly lowers parathyroid hormone (PTH) levels by increasing sensitivity of calcium sensing receptor on chief cell of parathyroid gland to extracellular calcium. Also results in concomitant serum calcium decrease  Indicated for hypercalcemia with parathyroid carcinoma. Do not lower Calcium too much  Serum calcium reduction may cause lowered seizure threshold, paresthesia, myalgia, cramping, and tetany;
  • 65. Criteria for Surgery – Primary hyperparathyroidism  Serum total calcium level >12 mg per dL (3 mmol per L) at any time  Hyperparathyroid crisis (discrete episode of life-threatening hypercalcemia)  Marked hypercalciuria (urinary calcium excretion more than 400 mg per day)  Nephrolithiasis  Impaired renal function  Osteitis fibrosa cystica  Reduced cortical bone density (measure with dual x-ray absorptiometry or similar technique)  Bone mass more than two standard deviations below age-matched controls (Z score less than 2)  Classic neuromuscular symptoms  Proximal muscle weakness and atrophy, hyperreflexia, and gait disturbance  Age younger than 50
  • 66.  A 66-year-old asymptomatic woman is evaluated for a serum calcium level of 11 mg/dL detected during a routine screening examination. Subsequently, the ionized calcium level is 5.7 mg/dL, and the intact parathyroid hormone level is elevated.  Which of the following is the most appropriate next step in the management of this patient?  ( A ) Parathyroidectomy  ( B ) Sestamibi scan of the parathyroid glands  ( C ) Creatinine clearance and 24-hour urine calcium excretion  ( D ) Chest radiograph  ( E ) Estrogen replacement therapy
  • 68. Remember!  “If suspecting Prolactin secreting adenomas or while working up the cause of hyperprolactinemia, the LEVEL OF PROLACTIN is the best clue to the possible etiology of hyperprolatinemia”  Prolactin level usually less than 100 with drugs or stress!
  • 69. Prolactinomas  Microademomas  prolactin levels usually > 100  Macroadenomas  Prolactin levels usually very high 500 to 1000. Be aware of HOOK Effect.  MRI is the next test if high prolactin level encountered and cannot be explained by meds/ stress. MRIs do not r/o small microadenomas (<3mm)  For microadenomas/ non invasive macroadenomas  use Bromocriptine ( dopamine agonist)  Consider surgery for those patients not responding to or intolerant of dopamine agonists.  If Prolactin Producing Macroadenomas with mass effect  Try bromocriptine first  if no response, Surgery ( trans sphenoidal resection)
  • 70. Pituitary Adenomas  In deciding how to approach INCIDENTALLY discovered pituitary tumors, first rule out Function  R/O functioning adenomas first  prolactin level, Dexamethasone suppression test, ACTH level, TSH and IGF 1 level depending on the clinical features.  Except Prolactinomas, all other functioning adenomas are treated primarily by surgery  If Non functioning adenomas  Surgery indicated only if mass effects  Visual field defects , Headaches and Hypopituitarism  Follow patients with non functioning microadenomas and small, noninvasive macroadenomas with a 6-month MRI scan and yearly thereafter  If there is no evidence of enlargement after 3 to 5 years, then continue to follow patients clinically.
  • 71.  A 25-year-old woman is evaluated because of a 9-month history of weight gain, fatigue, muscle weakness, and depression. She bruises and bleeds easily, and her menses have been irregular. Her medical history and family history are unremarkable. On physical examination, she is 157 cm (62 in) tall and weighs 74 kg (164 lb). Blood pressure is 160/95 mm Hg and pulse rate is 84/min. She has a rounded, plethoric face. Her supraclavicular fat pads are notably full, and she has a mildly enlarged dorsal fat pad (buffalo hump). She has several violaceous striae on the lower abdomen and bruises on the left arm from recent blood testing.  Laboratory Studies  Urine cortisol 318 μg/24 h  Morning serum cortisol 28 μg/dL  Morning plasma adrenocorticotropic hormone 45 pg/mL  After administration of dexamethasone, 8 mg orally at bedtime, the morning serum cortisol level is 3 μg/dL. Findings on chest radiograph are normal.  Which of the following tests should be ordered next?  ( A ) Low-dose (1 mg) overnight dexamethasone suppression test  ( B ) Magnetic resonance imaging scan of the pituitary gland  ( C ) Computed tomography scan of the adrenal glands  ( D ) Computed tomography scan of the lungs  ( E ) Inferior petrosal sinus sampling
  • 72. CARCINOID -Flushing, diarrhea, wheezing are clinical features -Ileal carcinoids commonly metastisizes to liver -Appendiceal carcinoids are most common. -If pt develops above symptoms on small doses of SSRIs  suspect Carcinoid syndrome -Screening test  Urine 5-HIAA -Confirmatory test – Octreotide Scan -Rx – medically, octreotide for symptoms -Surgery is definitive if no metastases
  • 73. Hypoglycemia  Medication induced in Diabetics Surreptious insulin/ sulfonyl urea abuse Insulinoma Treatment
  • 74. INSULINOMA - Whipples Triad – the symptoms of hypoglycemia - C-peptide level.
  • 75.  A 38-year-old woman is evaluated while in a stuporous state. The patient is slender and nearly comatose. She responds minimally to a loud voice and sternal pain. On physical examination, the pulse rate is 110/min and blood pressure is 115/70 mm Hg. An accompanying friend informs you that the patient is a nurse and that she has had recent psychiatric problems. The blood glucose level is 14 mg/dL. Additional blood is drawn, and the patient is quickly resuscitated with intravenous glucose. A sulfonylurea screen is negative.Laboratory Studies  Serum calcium 9.5 mg/dL  Serum insulin 45 mU/mL  C-peptide 4.2 ng/mL (normal, 0.5 to 2.0 ng/mL)  Proinsulin 0.6 ng/mL (normal, 0 to 0.2 ng/mL)  Which of the following is the best explanation for these findings?  ( A ) Solitary insulin-producing pancreatic islet cell tumor  ( B ) Surreptitious insulin use  ( C ) Surreptitious metformin use  ( D ) Multiple endocrine neoplasia type 1  ( E ) Multiple endocrine neoplasia type 2A
  • 76.  The differential diagnosis of fasting hypoglycemia associated with hyperinsulinemia includes insulinoma, surreptitious insulin use, and oral sulfonylurea ingestion.  Surreptitious insulin use is associated with low serum C-peptide and proinsulin levels.
  • 78. Congenital Adrenal Hyperplasia 21 Hydroxylase deficiency Increased 17 hydroxy progesterone, increased adrenal androgens. Low cortisol Ambiguous Genitilia in Congenital form Needs Mineralocorticoid and Glucocorticoid replacement – prevent salt wasting and hypotension Adult onset CAH is possible in mild deficiency – can be confused with PCOS
  • 80. Conns Syndrome Primary Hyperaldosteronism – Hypertension, Hypokalemia and Alkalosis PAC/ PRA ratio best screening test CT scan to r/o adrenal mass after biochemical evidence. Rx – medically with Spironolactone Surgery if tumor > 4cm or suspicious features of adrenal cancer or if refractory HTN despite maximal medical therapy
  • 81. Cushings Syndrome Etiology :Adrenal Cushings Pituitary Cushings Ectopic Cushings ( Small cell ca)  Diagnosis : Overnight Low dose (1mg) dexamethasone suppression test, Urinary 24 hr free cortisol, High dose (8mg) dexamethasone suppression test, ACTH level. CT abdomen if Adrenal cushing suspected Pituitary MRI if Pituitary adenoma suspected
  • 82. False +ve Low Dose dexamathasone test Factors that can produce false-positive or false-negative dexamethasone suppression tests:  Alcohol, rifampin, phenytoin, and phenobarbital induce the cytochrome P450-related enzymes and enhance dexamethasone clearance  Cause False +ve  Hepatic failure retards dexamethasone clearance  cause false –ve.  In renal failure, serum cortisol may appear nonsuppressible by dexamethasone  Obesity and depression give false-positive results  Thiazolidinediones can give false-negative results
  • 83. Addisons disease/ Adrenal insufficiency -Random cortisol level -Cosyntropin Stimulation test – 1hr and 24hr stimulation tests -Steroid replacement  HYDROCORTISOME 100 Q8hrs + Mineralocortcoid
  • 85. Work-up  In all Adrenal incidentalomas, R/o functioning adenomas first  Do Low dose dexamethasone test (1mg) and Plasma metanephrines in all patients  Do PAC/PRA only if HTN is present or if serum potassium is low.  If Functioning adenomas – Rx is usually surgery ( except in Conns Syndrome where medical Rx can be tried first)
  • 86. Work-up  For non-functioning adenomas, further treatment depends on the size of adrenal mass.  < 4cm  low risk of cancer ( 2% risk) . Follow-up Ct scan at 6-12 months. If no change, no further follow up.  4cm-6cm  6% risk of cancer  Adrenalectomy or a Follow-up CT Scan at 6months. If no growth, just follow clinically. ( no more imaging)  >6cm  high risk of malignancy (25% cases). Rx with adrenalectomy
  • 87.  After a fall while horseback riding, a 37-year-old woman is evaluated with an emergency CT scan of the abdomen. The CT scan shows no evidence of a ruptured spleen, but shows a 2.5-cm right adrenal mass. The patient’s medical history, including review of systems, is normal. Findings on physical examination, including blood pressure, are unremarkable. Plasma glucose level, serum electrolyte levels, and renal function are normal.  What is the most appropriate next step in the management of this patient?  ( A ) Surgical removal of the mass  ( B ) MRI of the adrenal glands  ( C ) Fine-needle aspiration biopsy of the mass under CT guidance  ( D ) Repeated CT scan in 3 to 6 months  ( E ) 24-Hour urinary catecholamines, metanephrine, and cortisol
  • 88.  35-year-old man with a 10-year history of type 1 diabetes mellitus is evaluated because of recent onset of morning hyperglycemia. For the past 10 days, his morning blood glucose levels ranged from 220 to 300 mg/dL. He has also experienced nightmares recently.  Which of the following best explains this patient’s morning hyperglycemia?  ( A ) Diabetic nephropathy  ( B ) Undertreatment with insulin  ( C ) Overtreatment with insulin  ( D ) Diabetic neuropathy
  • 89. KEY POINT SOMGYI EFFECT Nightmares are a clue and signifies a drop in blood glucose to low levels Next step  reduce the dose of pre-dinner insulin